Está en la página 1de 95

Universidad La Salle.

Facultad Mexicana de Medicina


Curso de Extensión Universitaria para la Preparación del Examen Nacional para
Aspirantes a Residencias Médicas.
Examen de Ginecología y Obstetrícia. 2018

1.- Se trata de femenino de 36 años nulípara tras 2 años de relaciones sexuales sin
contracepción, refiere que desde hace 1 año presenta dismenorrea, dispareunia y
sangrado vaginal intermenstrual. El diagnóstico más probable de la paciente es:

a) Insuficiencia luteínica.
b) Enfermedad inflamatoria pélvica.
c) Dismenorrea funcional.
d) Endometriosis.

La endometriosis consiste en la aparición y crecimiento de tejido endometrial fuera


del útero, sobre todo en la cavidad pélvica como en los ovarios, detrás del útero, en los
ligamentos uterinos, en la vejiga urinaria o en el intestino. Es menos frecuente que la
endometriosis aparezca fuera del abdomen como en los pulmones o en otras partes
del cuerpo.

La endometriosis es una enfermedad relativamente frecuente, que puede afectar a


cualquier mujer en edad fértil, desde la menarquia hasta la menopausia, aunque
algunas veces, la endometriosis puede durar hasta después de la menopausia. La
endometriosis altera la calidad de vida de las mujeres que la padecen, afectando a sus
relaciones de pareja, familiares, laborales y de reproducción.

Síntomas

Los síntomas clásicos son la dismenorrea, dolor pélvico, dispareunia, sangrados


intermestruales y en muchos casos, esterilidad.

El dolor no tiene que ver con el tamaño y la severidad de la lesión; generalmente


cuanto menor es la lesión mayor dolor produce. El dolor se agrava con las
menstruaciones y en los casos en que la lesión ocupa el fondo de saco de Douglas,
puede dar dispareunia. Existe un aumento de la PGF2 alfa y PGE2 y un aumento de
las contracciones uterinas que podría deberse a un depósito de endometrio en la
cavidad peritoneal.

La esterilidad debido a la endometriosis podría deberse a distintas causas de acuerdo


a la severidad de la patología. En los casos de endometriosis severa puede haber un
factor tuboperitoneal con adherencias y alteración en la anatomía de la pelvis que
interfiera con el transporte del esperma y el óvulo. En los casos de endometriosis leve
hay varios mecanismos propuestos que justifican su relación con la infertilidad:
foliculogénesis alterada, fase lútea inadecuada, fagocitosis espermática, mala calidad
ovocitaria, embriotoxicidad y alteración a nivel de la implantación.. La producción de
prostaglandinas por el endometrio ectópico puede afectar la motilidad tubaria, la
foliculogénesis y la función del cuerpo lúteo. Puede haber un aumento de la activación
de los macrófagos peritoneales en la endometriosis que cause la fagocitosis de los
espermas o la secreción de citoquinas que pueden ser tóxicas para el embrión. Según
algunos investigadores habría un 60% de las mujeres con endometriosis que
presentan un síndrome de Folículo Luteinizado no roto (LUF) en el cual el folículo no
se rompe en la ovulación y el óvulo queda atrapado.

Referencias bibliográficas

Ruiz V. Endometriosis y fertilidad. Ed. Acosta y Warman, pp. 99

El-Eoley, et al. Danazol but not ginadotropin releasing hormone agonists suppresses
autoantibodies in endomeriosis. Fertil Steril ; 54:725

Acosta AA. Buttram VC Jr. Besch PK, Malinak LR, Van Der Heyden J. A.proposed
classfication of pelvic endometriosis. Obstet Gynecol López VH. Tratamiento médico-
quirúrgico de la endometriosis..

2.- Se presenta en su consultorio paciente de 40 años de edad refiriendo mastalgia


que es más severa antes de la menstruación. A la palpación hay nodularidad excesiva,
hiperestesia y áreas quísticas que la paciente refiere disminuyen en tamaño después
de la menstruación. El diagnóstico más probable de esta paciente es:

a) Mastopatía fibroquística

b) Fibroadenomas

c) Papiloma intraductal

d) Cáncer de mama
Es el trastorno benigno de la mama más frecuente y consiste en un aumento del tejido
mamario, especialmente en las zonas superiores y externas de las mamas, hacia las
axilas, que las hace más densas.

La mastopatía fibroquística suele presentarse en ambas mamas, aunque puede ser de


diferente intensidad en una que en otra.

Puede presentarse a cualquier edad después del inicio de la menstruación, pero es


más probable que aparezca entre los 30 años y la menopausia. Raramente se
presenta más tarde de esa edad.

El origen de este trastorno es funcional y responde a desequilibrios de las hormonas


sexuales femeninas y puede condicionar la aparición de quistes mamarios.

Los síntomas pueden fluctuar de leves a severos en una mastopatía fibroquistica


mamaria, se acentúan típicamente antes de cada período menstrual y desaparecen
inmediatamente después.

Los síntomas abarcan:

 Consistencia de protuberancias , irregular y densa del tejido mamario


o generalmente más notoria en la parte superior externa de la mama
 Molestia en las mamas
o generalmente en ambas mamas
o puede ser persistente o puede aparecer y desaparecer
 Sensación de llenura en las mamas
 Sensibilidad y dolor sordo e intenso
 Sensibilidad y edema premenstrual
 Secreción ocasional del pezón

Bibliografía:

 Jones III HW, Wentz AC. Tratado de Ginecología de Novak. Editorial


Interamericana-McGraw Hill. Undécima Edición
 DiSaia-Creasman. Oncología Ginecológica Clínica. Editorial Mosby.
 Van Dinh T. Sumario de Patología Ginecológica. Editorial La Prensa Médica
Mexicana. 1
 Pernoll ML. Diagnóstico y Tratamiento Ginecoobstétricos. El Manual Moderno.
México.
3.- Femenino de 19 años, gesta 1, tuvo un parto con un producto único masculino de
3,600 g. la calificación de Apgar en el RN fue de 9 al primer minuto y 9 a los 5 minutos.
La revisión de sus registros de trabajo de parto mostró que tuvo ruptura de
membranas 7 horas antes del parto. 40 horas después del parto la paciente presenta:
temperatura 38.2 ° C, FC 105 x’, TA 110/70, FR 16x’; E.F. dolor leve a la palpación del
útero, las mamas se encuentran sin eritema, ni dolor a la palpación, no dolor a la
compresión de las pantorrillas. De los siguientes pasos es el más indicado antes de
iniciar terapia con antibióticos es:

a) Hemocultivo
b) Cultivo de secreción vaginal
c) Examen general de orina y cultivo
d) Legrado por aspiración

El vaciamiento incompleto ocasiona orina residual, distensión vesical excesiva y


estasis, además del cateterismo intermitente con sonda vesical durante el trabajo de
parto. Por lo tanto, la vejiga en el puerperio está predispuesta a infecciones. El dolor
leve a la palpación del útero puede ser normal en el puerperio y no se debe suponer
de inmediato endometritis puerperal. Cuando se sospecha endometritis, los cultivos de
secreción vaginal tienen poca utilidad porque se encuentran los mimos
microorganismos que en mujeres puérperas sanas. Los hemocultivos son apropiados
para la valoración diagnóstica de la fiebre puerperal, pero no son el paso inicial. La
espirometría por incentivo se utiliza en el posoperatorio de inmediato para fomentar la
expansión pulmonar y disminuir las atelectasias. El legrado uterino se utiliza para
tratar la hemorragia

Morgan M, Siddighi S. Ginecología y obstetricia, National Medical Series. 5° edición.


Mc Graw Hill. Pp. 29.

4.- Se trata de paciente femenino de 34 años de edad que inicia tratamiento con
sulfato de magnesio por presentar eclampsia, se presentan signos de sobre dosi-
ficación. El antídoto se debe emplear en esta paciente es:

a) Gluconato cálcico
b) Nitroprusiato.
c) Simpaticomiméticos.
d) Carbonato sódico.
NIVEL DE PRIMER CONTACTO (ATENCION PRIMARIA)

Se debe instruir a todas las embarazadas que deben acudir inmediatamente a un


centro de salud en cualquiera de los siguientes casos:

 edema que se desarrolla rápidamente (en pocos días)


 cefalea severa y persistente
 dolor en la región abdominal superior
 visión borrosa

Se debe realizar la medición de la presión arterial y un análisis de orina para la


detección de proteinuria a las mujeres que acudan a centros de salud presentando
estos síntomas.

Convulsiones

Si se asiste a una mujer con eclampsia en un centro de atención primaria,

1. deben mantenerse las vías respiratorias permeables;

2. se debe colocar a la mujer de costado (posición decúbito lateral izquierda) para


evitar la aspiración del vómito u otras secreciones;

3. si es posible, se debe establecer una vía intravenosa;

4. se debe administrar sulfato de magnesio.

Monitoreo de la administración de sulfato de magnesio: Durante el tratamiento con


sulfato de magnesio, se recomienda realizar un control cada 4 horas, como mínimo,
para detectar la presencia de: Reflejo rotuliano, frecuencia respiratoria superior a 16
por minuto, volumen de orina >100 ml en las 4 horas previas.

- Sobredosis de sulfato de magnesio: Todo centro de salud que utilice sulfato de


magnesio debe disponer de ampollas de gluconato de calcio (1 g) como antídoto para
la sobredosis de dicho fármaco.

 Se sugiere medir la presión arterial y administrar antihipertensivos según


corresponda.
 Convulsiones recurrentes: en caso de convulsiones recurrentes, se administran
otros 2 a 4 g de sulfato de magnesio por vía IV en el lapso de 5 minutos, tanto
para el régimen IM como el IV; la dosis se determina en función del peso de la
paciente.

El sulfato de magnesio es un fármaco usado en el control de las convulsiones


eclámpticas, para suprimir o controlar las contracciones uterinas sean estas
espontáneas o inducidas, y como broncodilatador luego del uso de beta agonistas y
agentes anticolinergicos. También tiene indicación como terapia de reemplazo en la
deficiencia de magnesio, como laxante para reducir la absorción de tóxicos del tracto
gastrointestinal. El sulfato de magnesio está ganando popularidad como tratamiento
de inicio en el manejo de algunas arritmias, particularmente en Torsades de Pointes, y
en arritmias secundarias a sobredosis de antidepresivos tricíclicos o toxicidad
digitálica. Esta también considerada clase Ila (probable beneficio) para la fibrilación
ventricular refractaria y la taquicardia ventricular, luego de la administración de dosis
de lidocaina y bretilio.

FARMACODINAMIA

El sulfato de magnesio tiene la capacidad de alterar la excitabilidad de la fibra


miometrial, afecta el acoplamiento excitación – contracción y el proceso mismo de
contracción, inhibe la entrada de calcio al sarcoplasma y reduce la frecuencia de los
potenciales de acción. Inhibe también la liberación de acetilcolina. Por ser estas
acciones comunes en las fibras musculares se pueden ver afectadas también la
musculatura voluntaria e incluso las fibras miocárdicas. (1)

Bibliografía.

Graves C. Fármacos que contraen o relajan el útero. En: Hardman J, Limbird L,


Molinoff P, Ruddon R, Goodman A, eds. Goodman & Gilman. Las Bases
Farmacológicas de la Terapéutica. 9 ed. México DF: McGraw-Hill Interamericana; . pp.
1012-3.

5.- Se trata de femenino de 56 años de edad refiere aumento de volumen a nivel


abdominal, con predominio de hemiabdomen inferior con la siguiente
sintomatología: plenitud, estreñimiento, se acompaña de USG pélvico que demuestra
imagen quística en ovario derecho de 15 por 15 cms. El diagnóstico más probable es:

a) Teratoma quístico.

b) Disgerminoma.

c) Endometrioma.

d) Cistadenoma seroso.

Los Tumores de Ovario son una patología frecuente dentro del contexto de la
patología femenina. Por esta causa consultan un grupo elevado de mujeres, tanto las
consultas de ginecología como las de Cirugía propiamente dicha. Las edades oscilan
desde las tempranas hasta las ya avanzadas, siendo el riesgo de degeneración
maligna muy variable y relacionado con le edad. La experiencia de la clínica revela la
alta incidencia de tumores de ovario en la etapa del climaterio, comprendida entre los
35 y 65 años de edad 1.

El cistoadenoma seroso de ovario (CSO) es un tipo de tumor derivado del epitelio


superficial (celómico), formado por áreas quísticas. El cistoadenoma seroso de ovario
es el tumor más frecuente de aquellos que provienen del epitelio celómico superficial.
Hay tumores pequeños macroscópicamente y tumores masivos que ocupan toda la
pelvis e incluso la cavidad abdominal. Estas frecuentes neoplasias quísticas
uniloculares están tapizadas por células epiteliales altas, cilíndricas y ciliadas, llenas
de un líquido seroso claro y de superficie lisa con abundantes vasos. Las variedades
benigna, limítrofe y maligna representan, en conjunto, 30% aproximadamente de todos
los tumores del ovario. El riesgo de presentar tumores epiteliales se incrementa con el
paso de la edad, ya que pese a que la declinación de la función ovárica marca el
envejecimiento gonadal progresivo, el ovario humano nunca pierde su capacidad para
generar tumores. Por lo general, cuando es detectado, su tamaño es grande, en donde
la imagenología puede ayudarnos a considerar su diagnóstico

1. Capítulo 22 Tumores Benignos de Ovario. En: Novak ER, Jones G., Jokes HW.
Tratado de Ginecología. 9 ed. Ciudad de la Habana. Editorial Científico
Técnica;.p.432 – 66.
2. MedlinePlus Enciclopedia Médica en Español: Quistes Ováricos. Disponible en:
http://vsearch.nlm.nih.gov/vivisimo/cgibin/querymeta?v%3Aproject=medlineplusspanis
h&spell=spell&query=Quistes+Ov%C3%A1ricos Acceso: Actualizado 20/6/06.

Capítulo XL Tumores Ováricos En: Llusiá Botella J, Núñez Clavero JA. Tratado de
Ginecología. Ciudad de la Habana. Editorial Científico Técnica. 1; T 3.1; p. 751 – 803.

6.- Se trata de paciente femenino de 23 años acude al servicio de ginecología por


referir ciclos opso-menorreicos, desde el inicio de su menarquia, en los últimos 5
meses ha incrementado 15 Kg. de lo que pesaba habitualmente. Exploración Física:
acné facial importante, así como bigote.
El diagnóstico más probable en esta paciente es:

a) Sx. De Asherman
b) Sx. Stein Leventhall
c) Sx. Amenorrea Galactorrea
d) Sx. Karman

Síndrome de Ovario Poliquístico (SOP) es uno de los más comunes trastornos


endocrinos que afectan a las mujeres alrededor del 5% al 10% de las mujeres en edad
reproductiva (12-45 años) y se piensa que es una de las principales causas de la
infertilidad femenina. Las características principales son la obesidad, anovulación
(dando lugar a la menstruación irregular) o amenorrea, acné, y las cantidades
excesivas o los efectos de androgénicos (masculinizantes) hormonas. Los síntomas y
la severidad del síndrome varían mucho entre las mujeres. Si bien las causas son
desconocidas, resistencia a la insulina, la diabetes y la obesidad están fuertemente
correlacionadas con el SOP.
Bulun SE, Adashi EY. The physiology and pathology of the female reporductive
axis. In: Kronenberg HM, Melmed S, Polonsky KS, Larsen PR, eds. Williams
Textbook of Endocrinology. 11th ed. Philadelphia, Pa: Saunders Elsevier;
2008:chap 16.

7.- Se trata de paciente femenino de 19 años de edad, es atendida en la sala de


urgencias gineco-obstétricas, Antecedente: cursa embarazo de 38 SDG. Exploración
Física: Se en trabajo de parto. Repentinamente presenta sangrado profuso
transvaginal y dolor abdominal intenso. A la paciente se le coloca un monitor fetal
externo. El tono uterino está incrementado, hay desaceleraciones variables y
ocasionales de la FCF que llegan a 90.
El manejo apropiado en esta paciente es:

a) Tocolisis con un agonista beta adrenérgico


b) Continuar el monitoreo de la madre y el producto
c) Amniotomía
d) Cesárea

El manejo de éstas pacientes es estricto y urgente:


1- Ingreso inmediato de la paciente en vigilancia a sala de partos.
2- Venopunción. Si existen signos de shock, instaurar preferentemente dos vías, una
central y otra periférica.
3- Solución de Ringer.
4- Monitoréo de signos vitales cada 15’.
5- Sonda vesical y monitoréo de diuresis. Diuresis de 50 ml/h asegura perfusión
periférica correcta.
Evitar diuresis inferiores a 30 ml/h.
6- Monitoréo contínuo de LCF.
7- Laboratorio de urgencia: hemograma, ionograma, urea y creatinina, coagulograma.
8- Sangre como mínimo 4U para reserva (concentrado de glóbulos rojos o sangre
entera).
9- Ecografía obstétrica para localizar y medir el hematoma y descartar otras causas de
sangrado.
Dependiendo del grado de desprendimiento realizar:
1- Desprendimiento leve: sin compromiso materno ni fetal y el embarazo es de
pretérmino, conducta expectante. Uteroinhibición y maduración pulmonar fetal.
2- Desprendimientos moderados a severos: si hay compromiso materno o fetal u óbito
fetal, terminar inmediatamente la gestación.
3- Independientemente de la vía del parto, es preceptiva la práctica de la amniotomía,
para reducir la presión intraamniótica y disminuir el paso de tromboplastina hística a la
circulación y la extravasación de sangre al miometrio.
Las principales complicaciones son:
1- Hipotensión, shock hipovolémico.
2- Coagulación intravascular diseminada.
3- Necrosis cortical y necrosis tubular aguda renal.

Obstetricia. Scwarcz, Sala, Duverges. 7ª edic. Edit. El Ateneo. (Biblioteca Fac.


Med. UNNE).

8.- Se trata de paciente femenino de 24 años de edad, Gesta 1, Para 1. Se refiere con
dos citologías “lesión de alto grado”, prueba de Schiller positiva y biopsia de cérvix que
demuestra carcinoma “In Situ”. La conducta más apropiada a realizar a la paciente es:

a) Conización.

b) Histerectomía total abdominal.

c) Histerectomía y salpingooforectomía bilateral.

d) Electrocauterización del cérvix.

La conización cervical es el tratamiento de elección en pacientes con cáncer


cervicouterino microinvasor y más si existe deseo de fertilidad. Asimismo, la
histerectomía extrafasciales un método adecuado en lesiones de 0.5 a 3 mm de
invasión.Además se propone que, para pacientes con lesiones de 3.1 a 5 mm de
invasión, a partir de la membrana inicial sinfactores de mal pronóstico como invasión
vascular y linfática, sean tratadas con histerectomía extrafascial, ya que en aquellasa
las que se realizó linfadenectomía pélvica, con este tipode lesión, no se encontró
metástasis a ganglios linfáticos.

Resultados del tratamiento en cáncer cervicouterino microinvasor en el Instituto


Nacional de Cancerología de México
1.- Mestwerdt G. Fruhdiagnose des Kollumkarzinoms. Zentralb Gynaekol, 1 ;69 :326.
2. - Morrow CP, Curtin JP. Surgery for cervical neoplasia. In Gynecologic Cancer
Surgery. New York, Churchill Livingstone, , p 472.3.

9.- Ante una paciente de 35 años que acude a consulta con antecedentes de G4 C2
A1, refiere que ha presentado durante el primero y segundo trimestres de su
embarazo manchado con frecuencia intermitente ,a las 34 SDG inicia con hemorragia
abundante, repentina e indolora, su principal sospecha diagnóstica es:

a) Coriocarcinoma
b) Ruptura uterina
c) Placenta previa
d) Desprendimiento grave de placenta normoinserta
PLACENTA PREVIA
DEFINICIÓN:
Es cuando la placenta se implanta sobre o muy cerca del orificio cervical interno y una
parte de la placenta precede a la parte fetal que se presenta.
INCIDENCIA:
Esta es difícil determinar ya que muchos casos pasan desapercibidos, sobre todo
cuando ocurren los abortos en embarazos tempranos.
La prevalencia varía de 1 en 100 a 1 en 850 nacidos vivos, pero solo el 20% total.
Etiología:
Edad avanzada, multíparas, paciente con cesáreas previas, paciente con aborto de
repetición, esto debido a las gestaciones previas.
TIPOS:
Inserción baja.- Es cuando el borde placentario se encuentra en el segmento inferior a
menos de 6 cm del orifico cervical interno.
Marginal.- Es cuando el borde placentario alcanza los márgenes del orificio cervical
interno.
Parcial.- Es esta la placenta cubre parcialmente el orificio cervical interno.
Total.- La placenta cubre la totalidad del orificio cervical interno aún con dilatación
cervical avanzada.
DIAGNOSTICO:
La característica es el STV de aparición brusca en forma indolora en el segundo o
tercer trimestre. Frecuentemente hay ausencia de dolor a actividad uterina que son
parámetros para hacer el diagnóstico.
La mayor incidencia de sangrado aparece a las 33-34 sdg.
ESTUDIOS DE GABINETE:
El estudio más utilizado es la ultrasonografía obstétrica.
TRATAMIENTO:
El manejo va a depender de factores como son:
Edad gestacional, magnitud del sangrado, si hay trabajo de parto, variedad de
placenta previa y complicaciones materna.

BIBLIOGRAFÍA:
1.- Waxler P, Gottesfeld KR. Early diagnosis of placenta previa. Obstet Gynecol
2.- Cabrero-Roura L. Riesgo elevado obstétrico. Ed. Masson
3.- Patrick J, Placenta Previa, Clinical Obst and Gynecology
4.- Chapman M, Furtenes ET, Significance of ultrasound in location of placenta in early
pregnancy Br J Obst Gynecol 197;86: 846.
57

10.- Se trata de femenino de 33 años con antecedentes patológicos de hipertensión


arterial crónica bien controlada tratada con IECAS, actualmente cursa con 7 semanas
de gestación, signos vitales dentro del parámetro normal y exámenes de laboratorio
sin alteraciones, se refiere asintomática. La conducta más apropiada a seguir es:

a) Suspender los IECAs dado el riesgo que presentan para el feto.


b) Mantener el tratamiento y asociar alfametildopa para disminuir los riesgos fetales de
los IECAs
c) Mantener el tratamiento y asociar hidralacina para disminuir los riesgos maternos de
los IECAs.
d) Mantener el tratamiento dado el buen control tensional.

El uso de IECA y ARAII durante el segundo y tercer trimestre de embarazo está


contraindicado, debido a que estos medicamentos inducen toxicidad fetal (descenso
de la función renal, oligohidramnios, retraso en la osificación del cráneo) y toxicidad
neonatal (insufi ciencia renal, hipotensión, hiperpotasemia).
En cuanto a su uso durante el primer trimestre de embarazo, un estudio publicado
en el año 20061 mostraba un incremento de la incidencia de malformaciones
congénitas, en particular malformaciones cardiacas, en niños nacidos de madres
expuestas a IECA durante el primer trimestre de embarazo en comparación con las
mujeres que no recibieron tratamiento antihipertensivo o que recibieron tratamiento
con otros medicamentos antihipertensivos. Estudios posteriores realizados no han
confirmado a día de hoy los resultados de este estudio. En lo referente a los ARAII, no
se dispone de estudios epidemiológicos analíticos apropiados, por lo que no se puede
descartar que exista el mismo riesgo que para los IECA.
A pesar de estas incertidumbres, el Comité de Medicamentos de Uso Humano
(CHMP) de la Agencia
Europea de Medicamentos (EMEA) ha recomendado prudencialmente evitar el uso de
IECA y ARAII durante el primer trimestre del embarazo.

Cooper WO et al. Major congenital malformations after fi rst-trimester exposure to ACE


inhibitors. N Engl J Med 2006; 354 (23): 243- 51.
(ref.: 2008/10, junio

11.- Se trata de paciente femenino de 19 años, con menarca a los 12 años ritmo
menstrual 45x4. Refiere vida sexual activa desde los 17 años con frecuencia de 4
veces por semana. A la exploración se encuentra acné intenso en la frente, mejillas y
mentón. Acude a consulta por que desea adoptar un tratamiento anticonceptivo por vía
oral, lo más adecuado es:

a) Norgestimato
b) Ciproterona
c) Gestodeno
d) Levonorgestrel

El efecto antiandrogénico específico del acetato de ciproterona actúa por inhibición


competitiva de la unión de la 5 - alfa - dihidrotestosterona con el receptor citosólico de
las células blanco, que disminuye la producción y la excreción de sebo y el aumento y
el desarrollo del vello.
Es un derivado de la 17 - alfa - hidroxiprogesterona que posee acción progestágena.
Su acción antigonadotrófica se suma a la del etinilestradiol. El acetato de ciproterona
no posee acción estrogénica sino un efecto antiestrogénico, y tampoco posee acción
nociva sobre la función de la corteza suprarrenal;

Indicaciones en la mujer: Manifestaciones de androgenización de grado severo, por


ejemplo, hirsutismo grave, alopecia androgenética de tipo grave, a menudo
acompañados por manifestaciones graves de acné y/o seborrea.
Indicaciones en el hombre: Atenuación del impulso en las desviaciones sexuales.
Tratamiento antiandrógeno del carcinoma de próstata inoperable.

BIBLIOGRAFÍA
1. Swift S. Current opinion on the classification and definition of genital tract prolapse.
Curr Opin Obstet
Gynecol 2002; 14: 503-7.
2. De Caro R, Aragona F, Herms A, Guidolin D, Bizzi E, Pagano F. Morphometric
analysis of the fibroadipose tissue of the female pelvis. J Urol 1998; 160: 707-13.
3. Gill E, Hurt W. Pathophysiology of pelvic organ prolapse. Clin Obstet Gynecol 1998;
25(4): 757-69.

12.- Se trata paciente femenino de 27 años de edad con deseo de un embarazo,


antecedentes de G3 A2 - P1 se le realiza una histerosalpingografía, se constata que
existe un síndrome de Asherman. Ello significa que se trata de:

a) Sinequias uterinas
b) Útero bicorne
c) Endometriosis en la trompa
d) Insuficiencia istmico cervical

El síndrome de Asherman es una enfermedad ginecológica rara que se caracteriza por


la presencia de sinequias (adherencias) intrauterinas que pueden ocasionar
amenorrea (ausencia de períodos menstruales regulares) e infertilidad.

En 1894 Heinrich Fritsch describe por primera vez la presencia de sinequias


intrauterinas de tipo postraumático, en una paciente que desarrolló una amenorrea
secundaria a un curetaje. Posteriormente en 1927 Bass informó de veinte casos de
atresia (oclusión de una abertura natural) cervical tras abortos inducidos, pero no fue
hasta 1948, cuando Joseph G. Asherman recopiló la información hasta entonces
existente y acuñó el nombre con el que se conoce actualmente a la enfermedad.

Asherman describió originalmente dos tipos diferentes de amenorrea secundaria, en


función de su etiología (estudio de las causas de las enfermedades): la amenorrea
traumática atrética, debida a estenosis del orificio cervical interno y la amenorrea
debida a adherencias intrauterinas. Posteriormente ambas entidades se agruparon en
una única entidad bajo el nombre de síndrome de Asherman.

Suele presentarse en mayor proporción tras dilataciones y curetajes uterinos de


repetición y sobre todo si se realizan durante el embarazo o si existe infección uterina
en el momento en el que se realizan estas intervenciones.

Las adherencias intrauterinas pueden producirse debido a cualquier factor que lleve a
una destrucción de las paredes del miometrio (capa muscular de la pared del útero).
Sin embargo, hay que distinguir entre factores predisponentes, siendo el principal de
ellos el embarazo y factores causales, entre los que se encuentran: traumatismos
uterinos, intervenciones quirúrgicas que afecten al útero, agentes físicos o químicos e
infecciones uterinas por tuberculosis o esquistosomiasis. En cualquier caso, el factor
más importante es el trauma uterino en el momento del parto o el puerperio.

El cuadro clínico es muy variable y las manifestaciones clínicas varían con el grado de
oclusión de la cavidad uterina y la severidad de las adherencias, pudiendo
presentarse: esterilidad cuando la oclusión de la cavidad uterina incluye porciones
proximales (más cerca de un centro, tronco o línea media) de las trompas de Falopio o
cuando las adherencias impiden la nidación del huevo; las pacientes presentan con
frecuencia amenorrea, oligomenorrea (disminución de la frecuencia de las
menstruaciones), dismenorrea (menstruación dolorosa) y abortos repetidos.

Hysteroscopic treatment of severe Asherman's syndrome and subsequent fertility.


Capella-Allouc S; Hum Reprod, 1999 May.

13.- En una consulta prenatal de rutina, una mujer de 28 años de edad, G5 P4, con 28
SDG, refiere que no ha sentido movimiento fetal durante los últimos 2 días. Su
embarazo ha sido complicado debido a que padece hipertensión crónica, para lo cual
es tratada con alfa-metildopa 2 veces al día. A la E.F. FU de 30cm, las maniobras de
Leopold demuestran que el feto se encuentra en situación transversa. Su TA es
145/85mmHg. No se encuentra latido cardiaco con el Doppler. De los siguientes el
paso más apropiado a seguir en el manejo es:

a) Realizar un test sin estrés


b) Amniocentesis
c) USG
d) Beta-HCG (cuantitativa)

Probable óbito:
Sintomatología y diagnóstico Signos funcionales: No se perciben movimientos
fetales por 12-24 horas. Disminución o ausencia de síntomas y/o signos como
nauseas vómito, hipertensión, albuminuria) Paraclínicos: *ecografía: diagnóstico
precoz y exacto: Doppler. *Radiología: hay 3 signos: +deformación del cráneo
+curvatura y torsión de la columna +presencia de gas en el feto *líquido amniótico:
puede estar meconiado, o sanguinolento Signos locales: en los senos hay secreción
calostral, sangrado leve y oscuro por vagina, el feto se vuelve blando a la palpación,
fetocardia (-), puede haber detención y/o disminución de la altura uterina, bajo peso
corporal, entre otros

Bibliografía: 1. OBSTETRICIA, Schwarcz R, editorial El ateneo, 2003. 2. Sociedad


española de ginecología y obstetricia, junio 2002. 3. OBSTETRICIA CLÍNICA, Llaca V,
edición 2000, capítulo 24; Pág, 315-316.
14.- Se trata de paciente femenino de 37 años de edad, G-3, C-2. Es ingresada a
hospital presentando cefalea, acúfenos, fosfenos y epigastralgia en barra con
embarazo de 34 semanas. E.F T/A 160/110, FC 84 x´, FR 18 x´, no presenta fiebre,
somnolienta, sin agregados cardioventilatorios, hepatalgia. F.U. de 25 cm. Producto
único vivo. FCF 110 lpm, genitales sin pérdidas ni modificaciones cervicales.
Laboratorio: hb 9.8 g/dl, plaquetas de 54 mil, TP 11 seg TPT 27, TGO 160 ng/dl TGP
160 ng/dl, hiperbilirrubinemia indirecta, albuminuria 300 mg/dl, Ácido Úrico de 8.1
mg/dl, creatinina de 1.5 mg/dl.
El diagnóstico más probable es:

a) Sx anticuerpos antifisfolípidos
b) Púrpura trombocitopénica trombótica
c) Sindrome de hellp.
d) Hígado graso

DEFINICIÓN:
• Es una complicación de la preeclampsia en la cual además de la Hipertensión Arterial
y proteinuria hay presencia de anemia hemolítica, enzimas hepáticas elevadas y
recuento bajo de plaquetas

MANIFESTACIONES CLINICAS:
• Malestar general, fatiga y molestias inespecíficas 90%
• Cefalea 70%
• Epigastralgia 64%
• Vómito 22%
• Fosfenos 15%
• Visión Borrosa 11%
• Acùfenos 3%
• Ictericia
• Anemia no explicada
• Oliguria
BIBLIOGRAFIA
Sibai baha, El síndrome HELLP. Universidad de Valencia, revista quincenal de
Obstetricia clínica y ginecología, octubre 2003.
V. Cararach, Síndrome de HELLP y Repercusiones maternas. X curso intensivo de
formación continuada materno fetal. Enero de 2003.
Toirac, Abelardo. Síndrome de Weistein HELLP Hospital Ginecoobstetrico Tamara
Bunke. Junio 2002

De la Fuente, David. Síndrome HELLP. Medicina Universitária 2003; 5 (19): 101 -9


Andrea G. Witlin, DO, Baha M. Sibai, MD. Diagnosis and Management of women
with Hemolysis Elevate Liver Enzymes, and Pletelet Count (HELLP) syndrome.
Hospital Physician. Febrero 1999.
CIFUENTES B, Rodrigo. Ginecología y obstetricia bas
15.- Se trata de paciente femenino de 19 años que acude a consulta refiriendo
presentar 6 meses sin regla. Se manifiesta preocupada por su sobrepeso, ha estado a
dieta y ha perdido 6 kg en 8 meses. Actualmente pesa 46 Kg. con talla de 1,65 mts.
Signos vitales dentro de sus parámetros normales. Se realiza prueba de embarazo con
resultado negativa. La causa más probable de la amenorrea de la paciente es:

a) Disgenesia gonadal.
b) Hipogonadismo hipogonadotropo.
c) Síndrome de ovario poliquístico.
d) Adenoma hipofisario.

El hipogonadismo femenino se caracteriza por un fallo gonadal debido a la alteración


del propio ovario o secundario a un fallo hipotálamo-hipofisario.
Este fallo de la gónada puede ocurrir en distintos momentos de la vida y por causas
diversas, lo que va a condicionar una presentación clínica diferente.
El ovario posee dos funciones relevantes: producir gametos femeninos (oogénesis) y
secretar hormonas esenciales en la regulación de la función reproductora y que
influyen en la diferenciación y el desarrollo de los órganos sexuales (hormonogénesis).
Los mecanismos celulares y moleculares responsables de la producción de oocitos y
hormonas por la gónada femenina son parcialmente independientes entre sí. Sin
embargo, ambas funciones son llevadas a cabo de modo concertado gracias a la
acción de un complejo sistema de control que implica interacciones múltiples entre el
hipotálamo, la hipófisis y el ovario.
Para la adquisición de la capacidad reproductora a partir de la pubertad se requiere
una adecuada formación e integración funcional durante las etapas tempranas del
desarrollo. Los elementos que componen este eje reproductor son: 1) el sistema
neuronal hipotalámico responsable de la producción del neuropéptido GnRH; 2) las
células gonadotropas de la hipófisis anterior que secretan LH (hormona luteinizante) y
FSH (hormona folículo estimulante), y 3) el ovario.
Hay que destacar que la formación de la gónada y de los elementos hipotálamo-
hipofisarios del eje gonadotropo se llevan a cabo de modo independiente en etapas
tempranas del desarrollo embrionario, produciéndose posteriormente su integración
funcional en circuitos de retroalimentación positivos y negativos, esenciales para una
correcta función reproductora.

FALLO GONADAL PRIMARIO O HIPOGONADISMO


HIPERGONADOTRÓFICO
Incluye a aquellos pacientes en los que la producción y acción de los esteroides están
reducidas. En los primeros 4 años de vida y a partir de los 9-10 años, la hipófisis, al
faltarle la retroalimentación negativa de estos esteroides, aumenta la producción de
gonadotropinas. Sin embargo, la elevación de las gonadotropinas no puede normalizar
la función gonadal.
Las causas pueden ser congénitas y adquiridas.

Etiología común a ambos sexos


Alteraciones congénitas del receptor de las gonadotropinas
Recientemente se han clonado y mapeado los genes de los receptores de
las gonadotropinas (LH-R y FSH-R) (20), estando ambos en 2p21, y se han
identificado diversas mutaciones y deleciones que conducen a una disminución de la
función de estos receptores, independiente de las gonadotropinas (hipogonadismo
hipergonadotropo); en otros casos se produce un aumento de la función de estos
receptores.
Mutaciones que ocasionan pérdida de la función del receptor de la LH
Hasta la fecha se han encontrado seis sustituciones de una base del gen del LH-R que
conducen a dos mutaciones nonsense o sin sentido que ocasiona un codón de parada,
y a cuatro mutaciones missense o mutaciones con error de sentido en los que cambia
un aminoácido en la proteína. La mayoría son homocigotos y se considera que el
trastorno es A-R. Clínicamente, existe una forma grave y una menos grave, lo que
tiene relación con la actividad del receptor. En las mujeres hay amenorrea, falta de
respuesta del ovario a la LH y respuesta normal a la FSH con desarrollo folicular
normal.
Mutaciones que ocasionan pérdida de la función del receptor de la FSH
Son más raras. Clínicamente las mujeres homocigotas presentan disgenesia ovárica e
hipogonadismo hipergonadotropo, mientras que las mujeres heterocigotas son
fenotípicamente normales.

BIBLIOGRAFÍA
1. Cañete R, Jiménez L. Ontogenia y diferenciación gonadal. Gónadas.
Actualizaciones en Endocrinología.
McGraw-Hill Interamericana. Capítulo 1, 2000.
2. Barrio R, Ezquieta B. Mecanismos genéticos de la diferenciación sexual: sus
alteraciones.
Gónadas. Actualizaciones en Endocrinología. McGraw-Hill Interamericana. Capítulo13,
2000.
3. De Roux N, Morel Y, Hardelin JP. Genetic anomalies of the gonadotropic axis. Rev
Prat
1999; 49 (12): 1277-1282.
4. Tena-Sempere M, Aguilar E, Pinilla L. Fisiología del eje hipotálamo-hipofiso-ovárico.
Gónadas.
Actualizaciones en Endocrinología. McGraw-Hill Interamericana. Capítulo 2, 2000.
5. Labarta JI y cols. Hipogonadismo hipergonadotrópico. En Pubertad normal y
patológica, 2. º curso de postgrado. Valladolid, 1996.
6. Veldhuis JD. Neuroendocrine mechanisms mediating awakening of the human
gonadotropic axis in puberty. Pediatr Nephrol 1996; 10 (3): 304-317.
7. Hopwood NJ. Pathogenesis and management of abnormal puberty. Spec Top
Endocrinol
Metab 1985; 7: 175-236.
8. Minagawa M, Yasuda T, Niimi H J. Spinal and femoral bone mass accumulation
during normal adolescence: comparison with female patients with sexual precocity and
with hypogonadism. Clin Endocrinol Metab 1996; 81 (3): 1248-1253.
9. Park KH, Lee SJ, Kim JY, Kim JY, Bai SW, Kim JW. A concomitant decrease in
cortical and trabecular bone mass in isolated hypogonadotropic hypogonadism and
gonadal dysgenesis. Yonsei Med J 1999; 40 (5): 444-9.
16.- Se trata de mujer de 31 años se envía de alta con diagnóstico de enfermedad
inflamatoria pélvica, regresa a los 15 días por malestar generalizado y dolor abdominal
intenso, con temperatura de 38.5 °c, datos de irritación peritoneal. El diagnóstico más
probable es:

a) Endometritis
b) Hidrosalpinx
c) Absceso tubo-ovárico
d) Ooforitis aguda

La enfermedad inflamatoria pélvica (EIPA) es un síndrome clínico caracterizado por la


infección del tracto genital superior que se produce casi siempre por vía ascendente
desde el cuello uterino. El impacto que la infección pélvica ejerce sobre la condición
física de la mujer va desde la infección asintomática o silente a una mayor morbilidad
que en algunos casos puede llegar hasta la muerte. Incluye una variedad de
condiciones inflamatorias que afectan el tracto genital superior. Los Centros de Control
de Enfermedades (C .D. E.) la definen como un síndrome agudo debido al ascenso de
microorganismos de la vagina o el cuello uterino al endometrio, trompas uterinas y en
ocasiones a las estructuras vecinas (ovarios, peritoneo y cavidad pelvianas).

En el momento actual se incluyen como principales agentes etiológicos de la E.I.P.A la


Neisseria gonorrhedae, las clamydias y los anaerobios. Otros microorganismos como
los microplasmas y los actinomices se están observando con frecuencia.
La presencia de anaerobios, así como de bacterias aerobias puede deberse a un
fenómeno de sobre infección secundaria. Hay autores que señalan que excepto para
el gonococo y la Clamydia trachormatis, no existen datos suficientes que permitan
afirmar que otras bacterias tengan un papel primario en la infección de unas trompas
sanas. Una vez alterada la integridad anatómica de la trompa, se produciría la
infección mixta o poli microbiana.

El absceso tubo ovárico es una formación inflamatoria que compromete el ovario y la


trompa y puede ser uni o bilateral. En este absceso las estructuras comprometidas
están infectadas y contienen pus. Este proceso inflamatorio es secundario a un
proceso infeccioso de la pelvis, habitualmente producido por gérmenes muy
patógenos, que llegan al tracto genital a través de una relación sexual, es
decir corresponde a una complicación severa de una enfermedad de transmisión
sexual.

Se caracteriza por aumento de volumen del ovario y trompa, los que se encuentran
adheridos entre sí producto de esta infección, además el proceso infeccioso se
extiende habitualmente a otras estructuras y órganos pelvianos, los que están muy
inflamados y adheridos í formando lo que se denomina plastrón.

El tratamiento se inicia médicamente con antibióticos de amplio espectro para


cubrir tanto gérmenes aeróbicos como anaeróbicos, generalmente requiere de
hospitalización para iniciar una terapia agresiva endovenosa con los antibióticos y
para monitorizar adecuadamente a la paciente, pues la infección produce
compromiso del estado general pudiendo llegar hasta la sepsis generalizada.

Diagnóstico

El cuadro clínico se sospecha cunado una paciente consulta por dolor abdominal
intenso, progresivo, fiebre y compromiso de su estado general, habitualmente en el
examen se encuentra un distensión abdominal y a la palpación del abdomen hay
dolor, y signo de blumberg positivo o irritación peritoneal. El Tacto vaginal demuestra
fondos de saco vaginales abombados y dolorosos y habitualmente el cuello del útero
lateralizado y doloroso a la movilización si el compromiso es unilateral, además de
palpar una masa para uterina irregular y sensible.

BIBLIOGRAFÍA.
1. Keit,L,G; Berger,G,S:On the causation of pelvic inflammatory disease. Am J
Obstet Gynecol 149-215,2002.
2. Muller,B,R;Allen,J,et al.Pelvic Inflamatory disease after
histerosalpingography.Brit J Obstet Gynecol,91-1181,1999.
3. Toth,A,O Leary,W,M: Evidence of microbial transfer by espermatozoo.Am J O
bstet Gynecol 59-556,2003.
4. Varela,R,et col:Abceso Tuboovárico,Acta médica Portuguesa ,p:537-542,Vol.
8,2001.
5. Sopper,D,E:Pelvic Inflamatory disease.Infections disease.Clin of North America
.831-840,vol 8;n 4.Dec 2003.

17.- Femenino de 25 años con antecedentes de G/2, P/1, C/1 acude al servicio de
consulta externa, refiere que presenta una secreción transvaginal bastante líquida, de
baja viscosidad, maloliente de color amarillo y gris espumoso.
El tratamiento de elección para esta entidad es:

a) Metronidazol 500 mg. VO c/12 por 7 a 10 días


b) Ampicilina 1g VO c/ 6 hrs.
c) Clotrimazol + Metronidazol 500 mg VO c/12 hrs. por 7 a días
d) Clindamicina 300 mg VO cada 12 hrs. x 5 días.
Referencias bibliográficas:
1. Secretaría de Salud. Norma Oficial Mexicana NOM -039-SSA2-2002, Para la
prevención y
control de las infecciones de transmisión sexual. D.O.F. 19 de Septiembre 2003.
2. Kettler H, White K, Hawkes S. Mapping the landscape for sexually transmitted
infections:
key findings and recommendations. Geneva, TDR (TDR/STI/ IDE/04.1).
3. CDC. Trends in Reportable Sexually Transmitted Diseases in the United States.
CDC,
National Report. 2004
4. Distribución de los casos nuevos de enfermedades por mes Estados Unidos
Mexicanos
2004. Sistema Único de Información para la Vigilancia Epidemiológica/Dirección
General
de Epidemiología/SSA
5. Aral S O. Sexual risk behaviour and infection: epidemiological considerations. Sex.
Transm.Inf. 2004;80:8-12

18.-Mujer de 26 años, es atendida en consulta en la clínica de displasias por


papanicolau con lesión NIC I. Antecedentes: menarca 14 años, ritmo 30x5
eumenorreica, inicio de vida sexual a los 15 años, 2 parejas sexuales, método de
planificación familiar oclusión tubaria bilateral, gestas 3 partos 3, cérvix con lesión
acetoblanca con extensión lineal de 2 cm. Se realiza biopsia de la lesión, en caso de
corroborarse el diagnóstico, el siguiente paso en el manejo de esta paciente es:

a) biopsia.
b) electrocirugía.
c) cepillado de canal.
d) ultrasonido endovaginal.
9.5.2 Las pacientes a quienes se les realizó citología cervical, cuyo resultado es
LEIBG (infección por VPH, displasia leve o NIC 1); LEIAG (displasia moderada y grave
o NIC 2 y 3) o cáncer deben enviarse a una clínica de colposcopía, para realizar
estudio colposcópico.
9.5.3 Si el resultado de la citología es LEIBG, la colposcopía es satisfactoria y sin
evidencia de LEIBG, se realizará control citológico en un año (Apéndice Normativo A)
9.5.4 Si la citología es de LEIBG, la colposcopía es satisfactoria y existe evidencia de
lesión, se debe tomar una biopsia dirigida.
9.5.4.1 Si la biopsia dirigida es negativa, se realizará nueva colposcopía para verificar
el diagnóstico y en caso necesario, tomar nueva biopsia dirigida y revalorar.
9.5.4.2 Si la biopsia dirigida es reportada como LEIBG se podrá dar tratamiento
conservador: criocirugía, electrocirugía o laserterapia (sólo si cumple con las
condiciones referidas en el Apéndice 1) o se podrá mantener a la paciente en
vigilancia en la clínica de colposcopía, con colposcopía y estudio citológico cada seis
meses, durante 24 meses.
Jueves 31 de mayo de 2007 DIARIO OFICIAL (Primera Sección)
9.5.4.3 Si la biopsia dirigida es reportada como LEIAG (Lesión Intraepitelial Escamosa
de Alto Grado) se realizará tratamiento conservador (electrocirugía o laserterapia). En
las mujeres posmenopáusicas, dependiendo de las condiciones anatómicas del cérvix,
se realizará tratamiento conservador en la clínica de colposcopía o tratamiento
quirúrgico (histerectomía extrafascial) en el servicio que corresponda.
9.5.4.4 Si la biopsia dirigida reporta cáncer microinvasor o invasor, la paciente se
transferirá a un Servicio o Centro Oncológico para su tratamiento correspondiente.
9.5.4.5 Si la citología reporta LEIBG y la colposcopía es no satisfactoria, se tomará
cepillado endocervical (Apéndice Normativo A)
9.6 En caso de colposcopía no satisfactoria, negativa a LEIBG y con cepillado
endocervical negativo, se continuará su control en la clínica de colposcopía en seis
meses, con colposcopía y citología.
9.6.1.1 Si el cepillado endocervical reporta LEIBG se tratará a la paciente como
LEIAG, con métodos conservadores escisionales.

Jueves 31 de mayo de 2007 DIARIO OFICIAL (Primera Sección)


Modificación a la Norma Oficial Mexicana NOM-014-SSA2-1994, Para la
prevención, detección, diagnóstico, tratamiento, control y vigilancia
epidemiológica del cáncer cérvico uterino.
Al margen un sello con el Escudo Nacional, que dice: Estados Unidos Mexicanos.-
Secretaría de Salud.
MODIFICACION A LA NORMA OFICIAL MEXICANA NOM-014-SSA2-1994, PARA LA
PREVENCION,
DETECCION, DIAGNOSTICO, TRATAMIENTO, CONTROL Y VIGILANCIA
EPIDEMIOLOGICA DEL CANCER CERVICO UTERINO.
19.- A 25-year-old woman presents with malodorus gray-wellow discharge. You take a
wet mount preparation and observe “Clue cells”. The agent of this infection and its
treatment is:

a) Gardnerella vaginalis / clindamycin


b) Gardnerella vaginalis / Ketoconazole
c) Trichomona vaginalis / metronidazole / treat the partner
d) Candida albicans / nistatin

Gardnerella vaginalis fue clasificada como una sola especie y fue establecida como
agente causal de la vaginosis (antes conocida como vaginitis inespecífica). El cuadro
clínico que presenta es caracterizado por una secreción blanca o blanco-grisácea que
se percibe generalmente después de la relación sexual con olor fétido aminado
(pescado). El diagnóstico certero es la base para evitar posibles complicaciones como
la enfermedad inflamatoria pelviana y las complicaciones del embarazo. El tratamiento
se basa principalmente en los fármacos como: metronidazol y clindamicina, debido a
su efectividad y espectro, pero como todos se deben emplear con adecuada prudencia
debido a su toxicidad. Además de que se deben corregir o modificar los factores
predisponentes, ya que esta patología va en aumento convirtiéndose por su frecuencia
en un problema de salud pública.

Bibliografía
1. Hernández F. Gardnerella vaginalis mobiluncus en la etiología de la vaginosis
bacteriana. Rev Costarricense Ciencias Médicas 1998; 19: 57-61.
2. Hansen EA. Gardnerella. Rev Ginecol 2005; 25: 99.
3. Espinosa I, Lorenzo M, Bentancourt A, Riverón Y, Romero M. Caracterización
bioquímica y antigénica de diferentes aislamientos de Gardnerella vaginalis.
Rev Cubana Invest Biomed 2005; 24: 22-7.
4. Taylor F. Vaginal flora morphotypic profiles and assessment of bacterial vaginosis in
women at risk for HIV infection.Infect Dis Obstet Gynecol
2004; 12: 121-6.

20.- Femenino de 26 años G-3, P-1, A-1 con 39 SDG por FUR. Reporta contracciones
uterinas que han sido regulares las últimas tres horas. Al examen encuentras que las
contracciones son cada tres minutos y duran 50 segundos y son firmes a la palpación.
Tuvo ruptura de membranas hace una hora y lo demuestras con papel de nitrazina. El
examen digital cervical demuestra una dilatación de 5 cm, con borramiento del 100% y
presentación en vértex en estación 0. De los siguientes criterios el más preciso para
decir que se encuentra en la fase activa del trabajo de parto es:

a) Borramiento cervical más de 90%


b) Duración de las contracciones de más de 30 seg
c) Dilatación cervical mayor de tres centímetros
d) Ruptura de membranas
FASES DEL TRABAJO DE PARTO

El trabajo de parto se divide en tres fases:

Fase 1 ó latente

Es llamado así al periodo que sirve para la preparación uterina del parto, ocurre al final
del embarazo y va hasta el inicio de las contracciones del trabajo de parto. Los
aspectos a destacar en este lapso es el reblandecimiento cervical, el aumento
importante en el número de receptores para oxitocina a nivel de las células
endometriales, un aumento sustancial en los puentes de unión y el número de
conexinas a nivel miometrial y por consiguiente una mayor sensibilidad a los agentes
uterotónicos.

Fase 2 ó activa

Es el lapso que representa el trabajo de parto activo, y se acepta que se inicie cuando
existen 3 cm de dilatación y las contracciones uterinas son aptas para producir un
avance en el trabajo de parto; se divide en tres periodos:
Primer periodo. Se inicia cuando las contracciones uterinas alcanzan la frecuencia,
intensidad y duración suficientes para causar borramiento y dilatación del cuello
uterino, y finaliza cuando éste se encuentra en completa dilatación.
El lapso de tiempo que dura es variable, pero se acepta como normal hasta diez horas
en primigrávidas y ocho horas en multigrávidas; pero independientemente de esto, se
debe considerar como adecuado si el borramiento y la dilatación cervical son
progresivos e ininterrumpidos.

Segundo periodo. Se inicia con una dilatación cervical completa y termina con la
expulsión del feto; tiene una duración variable, pero se acepta como normal una hora
en pacientes primíparas y 30 minutos en multíparas; y tiene como característica que
debe de ser progresivo e ininterrumpido.

Tercer periodo. Este comienza inmediatamente finalizada la expulsión fetal y termina


con la expulsión total de la placenta y las membranas corioamnióticas; a este periodo
se le conoce también como de “alumbramiento” y es el más corto de los periodos del
parto; como norma general se acepta que no debe de extenderse más allá de 10
minutos.
Existen algunos autores que incluyen un “cuarto periodo” dentro del trabajo de parto, el
cual abarca aproximadamente la hora posterior al alumbramiento, y comprende el
lapso de tiempo cuando ocurre la contracción y retracción de las fibras miometriales,
así como la trombosis de los vasos adyacentes, lo cual es un efectivo control de la
hemorragia del sitio de implantación de la placenta.

Fase 3

Este periodo es el que representa el regreso de la mujer a su estado previo al


embarazo, y se caracteriza por la involución uterina, la eyección láctea y por último la
restauración de la fertilidad; existen estudios que involucran en esta fase a la
endotelina-1 y a la oxitocina como substancias responsables de estos cambios
postparto.
PROGRAMA DE ACTUALIZACION CONTINUA PARA GINECOLOGÍA Y
OBSTETRICIA
PAC GO-1 Libro 3 Obstetricia 2005

21.- Se trata de femenino de 29 años de edad, la cual inicia con hiperemesis gravídica
de difícil control, así como sangrado trasvaginal. Se realiza el diagnóstico de mola
hidatiforme. El tratamiento inicial más apropiado en esta patología es:

a) Histerectomía total abdominal.

b) Legrado por aspiración.

c) Metotrexate y seguimientos radiográficos.

d) Legrado uterino instrumental

La enfermedad trofoblástica gestacional agrupa a diferentes entidades


interrelacionadas: mola completa, generalmente diploide con origen cromosómico
paterno, mola parcial generalmente triploide, tumor trofoblástico del lecho placentario y
coriocarcinoma, con tendencias variables a la invasión local y a las metástasis, cuyo
denominador común es la hipersecreción de hCG. El coriocarcinoma es diploide y
proviene de ambos progenitores, excluyendo probablemente su origen directo en la
mola completa. El tumor trofoblástico del lecho placentario está constituido por
trofoblasto mononuclear intermedio no conteniendo vellosidades coriónicas e
inmunohistoquimicamente caracterizado por expresar muchas de sus células hPL y
unas pocas hCG

Tratamiento

Hay que tratar las complicaciones como la hiperémesis, anemia, hipertensión, y


alteraciones electrolíticas, coagulopatías, alteraciones cardio-respiratorias y
preeclampsia, procediendo a evacuar la mola lo antes posible, con lo que se
producirán menos malignizaciones.

La evacuación del contenido uterino se realiza mediante dilatación, y legrado por


aspiración. Además se pauta profilaxis antibiótica y oxitócicos.3

La histerectomía, con la mola en su interior, está indicada en pacientes de edad


superior a 40 años o en mujeres con más de tres hijos, ya que en ambos grupos se ha
demostrado una mayor incidencia de malignización.
. Tras la cirugía, se mide la concentración de gonadotropina coriónica humana para
determinar si la extirpación ha sido completa. Si es así, el valor de esta hormona
vuelve a la normalidad, en unas 8 semanas, y se mantiene en esos valores. Si una
mujer a la que se le ha extirpado una mola queda embarazada, es difícil interpretar un
valor alto de gonadotropina coriónica humana, porque podría estar causado tanto por
el embarazo como por una parte de la mola que no se ha extirpado. En consecuencia,
a las mujeres a las que se les ha extirpado una mola se les recomienda no quedar
embarazadas durante un año. Las molas hidatiformes benignas no necesitan
quimioterapia, pero las malignas sí. Los fármacos que se usan para este tratamiento
son el metotrexato, la dactinomicina o una combinación de ambos.

6- Jones. Enfermedad Trofoblástica Gestacional: qué hemos aprendido en la última


década. Am J Gynecol Obstet
;162: 1286-1292.
8- Gonzalez Merlo et al. Protocolos de diagnóstico y tratamiento en Obstetricia y
Ginecología, Barcelona: editorial
Salvat, 7:35-45.

22.- Mujer de 23 años de edad, gesta 1, tuvo un parto vaginal espontáneo con un
producto con peso de 4,350 g. después de 5 minutos de tracción suave del cordón
umbilical se expulsó la placenta, que parece estar intacta. Se inició el masaje del fondo
uterino se administrará 20 unidades de oxitocina en 100 ml de solución Ringer lactato.
Después de una inspección cuidadosa del canal del parto se observa una laceración
de segundo grado y una laceración de 2 cm en la pared vaginal izquierda que se
intentó reparar. En la exploración física se encuentra un fondo uterino blando y
atónico. S.V.: temperatura 37.1°C, TA 164/92, FC 130x’, FR 18 X’. El tratamiento de
elección para esta paciente es:

(a) Oxitocina 10 unidades directas en goteo intravenoso

(b) Metilergonovina 0.2 mg IM

(c) Prostaglandina F 0.25 mg IM

(d) Legrado

Morgan M, Siddighi S. Ginecología y obstetricia, National Medical Series. 5° edición.


Mc Graw Hill. Pp. 28. La atonía uterina es la causa más común de hemorragia
puerperal. El masaje energético y la oxitocina diluida no han sido útiles para
interrumpir la hemorragia y por tanto el siguiente paso es agregar un fármaco
uterotónico. La metilergonovina está contraindicada porque la paciente se encuentra
hipertensa a pesar de la hemorragia intensa, el siguiente fármaco es la prostaglandina.
La administración de oxitocina no diluida, 10 UI por vía IV podría causar hipotensión
grave. La exploración manual podría ser apropiada si se sospecha laceración como
causa de hemorragia. El legrado es apropiado para la hemorragia puerperal tardía,
cuando se sospecha retención de los productos de la concepción.
23.- Se trata de paciente femenino de 32 años que cursa en éste momento con
diagnóstico de preclampsia leve, el fármaco de elección que se administra en esta
patología es:

a) Nifedipina.

b) Inhibidores de la enzima convertidora de angiotensina.

c) Clonidinas.

d) Alfametildopa.

• Prevenir complicaciones a corto plazo de las mujeres con PA elevada que


comprometa el bienestar fetal
• Cuando la PAS es mayor o igual a 150 mmHg y la PAD mayor o igual a 100
mmHg.

 El propósito es alcanzar cifras de TA alrededor de 140/90.


 La medicación antihipertensiva se reserva para los casos en que la PAD ≥ 100
mmHg.
 Se recomienda continuar el tratamiento antihipertensivo previo al embarazo,
exceptuando el uso de IECA.
 La alfametildopa y la hidralazina vía oral son los fármacos de elección dado su
uso extensivo con seguridad y eficacia y sin efectos colaterales para el feto
(excepto hidralazina en lupus).

• ALFA METILDOPA
500-2000 MG/DÍA
• HIDRALAZINA
50-200 MG/DÍA
• LABETALOL
100-400 MG/DIA
• ATENOLOL
50-200 MG/DÍA
• NIFEDIPINA
10-30 MG/DÍA

1. Aagard K, Belfort M. Eclampsia: Morbility, mortality, and management. Clin


Obstet Gynecolol. 2005; 48: 12-23.
2. Oyarzún E. Síndrome hipertensivo del embarazo en Oyarzún E. Ed. Embarazo de
alto riesgo. Ediciones Universidad Católica de Chile. Santiago. 1997: 157-
175.
3. Roberts J, Redman C. Pre-eclamsia: More than pregnancy induced hypertens
24.- Una mujer de 27 años G/4, P/0 con 6 SDG acude a su primera visita prenatal. Su
historia obstétrica pasada es importante porque tiene tres pérdidas de producto en el
segundo trimestre. Refiere que en las tres ocasiones al presentarse al hospital
presentaba dilación cervical completa. No recuerda haber tenido contracciones
dolorosas. Niega antecedentes médicos y quirúrgicos. El examen físico es normal
incluyendo un examen pélvico que muestra un cervix largo y cerrado. Después de una
larga discusión con la paciente ella pide que se le practique un cerclaje durante este
embarazo. El momento más apropiado para realizarlo es:

a) Inmediatamente
b) 13 a 16 semana
c) 24 a 28 semanas
d) 32 a 36 semanas

El cerclaje cervical tiene sus indicaciones en la profilaxis y tratamiento de la


incompetencia cervical.
La incompetencia o insuficiencia cervical representa un 10% de las causas de parto
pretérmino y está asociada a una importante morbimortalidad neonatal.
Las modificaciones cervicales en el segundo trimestre de gestación son causa de
parto prematuro y pueden deberse a:
1) Incompetencia cervical.
2) Pérdida de tejido conectivo tras una cirugía cervical (conización).
3) Defectos congénitos como la hipoplasia cervical tras exposición a dietilestilbestrol.
4) Infección intrauterina. Hasta un 51.5% de las pacientes con clínica compatible con
incompetencia cervical enmascaran un cuadro de infección intraamniótica subclínica.

Diferenciamos tres tipos de cerclaje:


1. El cerclaje se considera profiláctico o electivo (o primario) cuando se realiza de
forma electiva por historia previa de incompetencia cervical antes de evidenciar
cambios en el cervix y generalmente suele realizarse entre las 13 y 16 semanas de
gestación.
2. El cerclaje terapéutico secundario que se realiza tras la detección, en el
seguimiento obstétrico, de modificaciones en el cérvix antes de las 26 semanas de
gestación. Se realiza en pacientes con un riesgo potencial de parto pretérmino.
3. El cerclaje terapéutico terciario, en caliente, de rescate o “emergent cerclage”
que se realiza en pacientes que presentan la membrana amniótica visible a través del
orificio cervical externo o en vagina.

GUIA CLÍNICA:
INDICACIONES DEL CERCLAJE
Unitat de Prematuritat. Servei de Medicina Maternofetal.
Institut Clínic de Ginecologia, Obstetrícia i Neonatologia, Hospital Clínic de Barcelona
Responsables del protocolo: T.Cobo, M. López, M. Palacio
Creación: 24/01/07
Modificaciones: 05/09/07
Última actualización: 17/01/10
25.- Mujer de 26 años, con tumor anexial de 6 cm líquido, dolor abdominal, fiebre,
leucorrea, con historia de cervicovaginitis de repetición y dispareunia crónica, última
menstruación hace una semana. El diagnóstico más probable es:

a) Cistadenoma
b) Embarazo ectópico
c) Quiste de ovario
d) Enfermedad pélvica inflamatoria

La EIP puede cursar con los siguientes síntomas:

 Dolor abdominal bajo (incluyendo dolor anexial, dispareunia). Es el síntoma


más frecuente (95%)
 Aumento del flujo vaginal, flujo de características anormales (74%)
 Sangrado anormal (intermestrual, poscoital) (45%)
 Síntomas urinarios (35%)
 Vómitos (14%)
 Es posible la ausencia de síntomas

Y en ella podemos encontrar estos signos:

 Dolor a la movilización del cuello, dolor anexial en la exploración vaginal


bimanual (99%)
 En el examen con espéculo observamos cervicitis y descarga endocervical
purulenta (74%)
 Fiebre (> 38º C) (menos del 47%).
 Masa pélvica: sugiere abceso tuboovárico (ATO)
 Peritonitis

CRITERIOS CLINICOS PARA EL DIAGNOSTICO DE SALPINGITIS

a. Dolor abdominal con o sin rebote.


b. Sensibilidad a la movilización del cérvix.
c. Sensibilidad anexial.

Los tres criterios anteriores son necesarios para establecer el diagnóstico, con uno o
más de los siguientes:

a. Extendido de Gram de endocérvix positivo, para diplococos gram


negativos intracelulares
b. Temperatura mayor de 38°C
c. Leucocitosis (mayor de 10.000 por c.c.)
d. Material purulento (positivo para leucocitos) en la cavidad peritoneal
obtenido por culdocentesis o laparoscopia.

Establecido el diagnóstico clínico de EPI, se debe hacer la definición del estado clínico
y anatómico de la patología pélvica:

a) No complicada (limitada a trompas u ovarios)


1) Sin peritonitis pélvica

2) Con peritonitis pélvica

b) Complicada (masa inflamatoria o absceso que compromete trompa (s) u ovario (s)

1) Sin peritonitis pélvica

2) Con peritonitis pélvica

Beigi RH, Wiesenfeld HC. Pelvic inflammatory disease: new diagnostic criteria and
treatment. Obstet Gynecol Clin Norh Am. 2003; 30 (4): 777 – 93
Center for Disease Control. Guidelines for treatment of sexually transmited diseases.
MMWR Recomm Rep. 2002 May 10;51(RR-6):1-78
Prodigy Guidance. Pelvic inflammatory disease. [Internet]. UK : NHS, Department of
Health; 2003. [Acceso 18 de Junio de 2005]. Disponible en:
Ross J. Pelvic inflammatory disease. Clin Evid. 2004 Dec;(12):2259-65.
Royal College of Obstetricians and Gynaecologists. Pelvic Inflammatory Disease.
Guideline nº 32. [Internet]. RCOG; Mayo 2003. [Acceso 18 de Junio de 2005].

26.- Se trata de paciente de 34 años que cursa con 39 SDG; a la exploración física
reflejos patelares hiperactivos, inquieta, se reportan cifras de TA 145/95, se realiza
laboratorio que reporta proteinuria 2+

El tratamiento más adecuado para esta paciente es:

a) reposo en cama

b) Sulfato de magnesio oral

d) Propranolol

e) Inducir el trabajo de parto

Interrupción del embarazo


La mayor parte de las guías de práctica clínica que contestan esta pregunta se basan
en estudios retrospectivos o recomendaciones de los comités de expertos (Nivel III/IV).
La interrupción del embarazo se considera la mejor opción de tratamiento para la
preeclampsia y, sin duda, lo es para la embarazada al prevenir la preeclampsia severa
y la eclampsia. Sin embargo, para el feto no siempre es la mejor opción. Dos estudios
observacionales retrospectivos mostraron que a las 37 semanas la terminación del
embarazo es la mejor opción, tomando en cuenta que si existen condiciones cervicales
favorables, la inducción del parto es una via segura y de bajo riesgo para el feto (Nivel
III). En embarazos pretérmino debe considerarse la prolongación del embarazo hasta
alcanzar el mayor peso y madurez fetal.
Atención conservadora de pacientes con preeclampsia leve
Cuando el embarazo cursa entre las 28 y 34 semanas, el manejo conservador puede
reducir las complicaciones propias de la prematuridad. La decisión debe basarse en la
estabilidad del estado materno y fetal, así como en la capacidad del centro hospitalario
para brindar atención optima a un recién nacido prematuro (Nivel III). Durante el
periodo del monitoreo ambulatorio se indica a la paciente que se realice en forma
externa una medición diaria de la presión arterial, y que acuda semanalmente a la
consulta para descartar evolución o agravamiento de la enfermedad (Nivel IV). No
existen estudios controlados que hayan determinado la magnitudde los riesgos
maternos y fetales durante el tratamiento conservador de la paciente con preeclampsia
leve.
Recomendaciones
La paciente con preeclampsia leve, seleccionada y clasificada después de su estancia
hospitalaria, puede atenderse en forma ambulatoria, con vigilancia semanal en la
consulta, como una medida efectiva y segura.
(Grado de recomendación B)
En las pacientes con embarazo ≥ 37 semanas y preeclampsia leve, la terminación del
embarazo es la mejor opción, valorando la inducción del parto como una medida de
bajo riesgo para el feto.
(Grado de recomendación C)
La atención conservadora del embarazo menor de 34 semanas, complicado por
preclampsia leve, mejora el pronóstico del feto.
(Grado de recomendación C)

Ginecol Obstet Mex 2010;78(6):S461-S525


Guías de práctica clínica
Diagnóstico y tratamiento de la
preeclampsia-eclampsia
Fecha de busqueda de informacion: marzo 2009
Fecha de elaboracion: septiembre 2009
Fecha de actualizacion: junio 2012
Institución responsable: Colegio Mexicano de Especialistas en Ginecología y
Obstetricia.
Coordinador del grupo
Dra. Maria Teresa Leis Marquez
Especialista en Ginecología y Obstetricia y Medicina Materno Fetal. Certificada por el
Consejo Mexicano de Ginecología y Obstetricia. Jefa de la Clínica de Medicina
Materno Fetal del Hospital Ángeles Lomas.
Coordinadora del Comité para la elaboración de las Guías de Práctica Clínica del
Colegio Mexicano de
Especialistas en Ginecología y Obstetricia, periodo 2008-2010. Miembro de la Junta de
Gobierno y del
Comité de Exámenes del Consejo Mexicano de Ginecología y Obstetricia.
Vicepresidenta para América del Norte de la Sociedad Iberoamericana de Diagnóstico
y Tratamiento Prenatal.

27.- Se trata de mujer de 18 años, acude al servicio de urgencias por presentar


sangrado transvaginal. Antecedentes: G2 - P1, cursa con embarazo de 9 semanas de
gestación. E.F. Sangrado transvaginal moderado, rojo rutilante. , cérvix posterior,
largo, blando, orificio cervical externo dehiscente, dolor a la movilización cervical,
rebote negativo. TA 110/70 MMHG FC 92X´FR 19 X´T 37 °C.
La conducta más apropiada a seguir en la paciente es:
a) hospitalización y reposo
b) realizar ultrasonido abdominal
c) realizar ultrasonido transvaginal
d) cuantificación HGC fracción beta

El examen ultrasonográfico transvaginal es fundamental para establecer la presencia


de un embarazo intrauterino o la falla temprana del embarazo y para excluir otras
causas de sangrado como embarazo ectópico o molar. Por lo tanto El ultrasonido
transvaginal en un aborto en evolución es el método de elección para saber qué tipo
de aborto está presentando la paciente.

1a Oxford Revisión sistemática (con homogeneidad*)de estudios controlados


aleatorizados. Raj Mohan Paspulati,2007

28.- Se trata de paciente femenino de 25 años, G.1 , en sala de expulsión en postparto


inmediato se obtiene producto de 4100 gr, APGAR 6/9, sangrado postparto
cuantificado en 1200 ml. A la EF paciente pálida diaforética con TA 90/60, FC 128x,
FR 24, T 37.2°C. La medida inmediata más apropiada para la paciente es:

a) canalizar segunda via, aplicar misoprostrol 1000 mcg rectales


b) canalizar segunda vía y aplicar solución cristaloide tibia a 30ml/kg
c) aplicar oxitocina 10 ui im + carbetocina 100 mcg iv
d) revisar canal del parto y aplicar solución coloide a 40ml/Kg

En caso de presentar sangrado transvaginal en el puerperio inmediato, se debe


determinar si existen signos sugestivos de choque como: 1) hipotensión (PAS <90
mmHg o disminución de la PAS mayor a 40 mmHg de su presión basal, 2) Presión
arterial media <60 mmHg, 3) FC >120 lpm, 4) Volumen urinario <0.5ml/kg/h, 5) llenado
capilar >3 segundos. En caso de existir signos sugestivos de choque se debe pedir
ayuda (código mater), asegurar la vía aérea y restitución de líquidos. En choque leve y
moderado se realiza la restitución con cristaloides a 30ml/kg o coloides a 20ml/kg.
Además de esto se debe valorar el uso de uterotónicos, estado neurológico del
paciente y control de la temperatura.
Diagnóstico y tratamiento del choque hemorragico en obstetricia. Guía de
Referencia Rápida: Guía de Práctica Clínica. México, CENETEC; Disponible
en: http://www.cenetec.salud.gob.mx/contenidos/gpc/catalogoMaestroGPC.html
29.- Mujer de 38 años, que acude a urgencias, por cefalea 8/10, acúfenos y fosfenos;
G3P2, embarazo de 19 semanas, cérvix cerrado formado posterior, sin perdidas
transvaginales, no refiere antecedentes de importancia
TA 145/100 y 140/100 a los 10 minutos, FC 87 lpm, FR 18 rpm, T 37°C.
Lab: EGO tira reactiva proteinas ++
La conducta terapéutica más apropiada en este caso es:

a) Ingreso a hospital, toma de exámenes de laboratorio y US obstétrico.


b) Iniciar tratamiento alfa metil dopa 250 cada 8 hs.
c) Iniciar Ácido acetil salicílico 80 mg por día
d) Iniciar tratamiento alfa metil dopa 250 cada 8 + hidralacina 30 mg cada 8 hs vía
oral

En el abordaje diagnóstico de la paciente con preeclampsia debe de realizarse


estudios auxiliares diagnósticos para el monitoreo y vigilancia de su evolución: a)
Vigilancia estrecha de presión arterial monitoreo, b) toma de laboratorios como BH, c)
Confirmar o descartar proteinuria mediante cuantificación de proteínas en orina de
24hrs, d) Ultrasonido obstétrico para evaluar crecimiento fetal y líquido amniótico si las
condiciones maternas lo permiten, entre otras cosas. Se recomienda establecer
diagnóstico de proteinuria cuando presenta uno de los siguientes: La cuantificación de
proteínas en una recolección de orina de 24 horas es mayor o igual a 300 mg. O
presencia de 1+ en tira reactiva en una muestra tomada al azar. (Utilizar solo si los
métodos cuantitativos no están disponibles).
Diagnóstico y tratamiento del choque hemorragico en obstetricia. Guía de
Referencia Rápida: Guía de Práctica Clínica. México, CENETEC; Disponible
en: http://www.cenetec.salud.gob.mx/contenidos/gpc/catalogoMaestroGPC.html

30.- Se trata de femenino de 31 años nuligesta pero con actividad sexual regular, sin
método de planificación familiar, con ciclos regulares, sin leucorrea, refiere
dispareunia profunda, sangrado intermestrual, dismenorrea secundaria y
ocacionalmente presenta urgencia urinaria.
El tratamiento más adecuado para esta paciente es:

a) Doxicilclina
b) Ceftriaxona
c) Leuprolide
d) Tibolona
Endometriosis
Tratamiento
Análogos de  GnRH por 6  meses (Leuprolide,  Nafarelina, 
Triptorelina, Goserelina.

• Disminuye la secreción de FSH y LH .
Danazol
ACO por 9 meses
Laparoscopia
Radical con HTA +SOB
¿Cáncer?
Células claras
Endometroide de ovario

31.- Una mujer embarazada, puede afectar al feto y hacerlo contraer lesiones
importantes durante el embarazo o al salir al exterior (atravesando el canal de parto),
sí la gestante se encuentra afectada de la siguiente patología:

a) Tricomonas
b) Gardenerella
c) Herpes genital
d) Gonococos

Herpes genital

La prevalencia de herpes simplex genital o tipo 2 (VHS-2) en mujeres embarazadas


varía entre 7 y 33% en distintas series. La prevalencia ha experimentado un sostenido
aumento durante los últimos años. Se estima que aproximadamente 1 a 3% de las
mujeres adquiere cada año la infección. En el caso de parejas discordantes, la tasa de
adquisición aumenta hasta 10 a 30% anual. La adquisición durante el embarazo es
~2%.

La transmisión al hijo ocurre principalmente cuando la mujer embarazada adquiere


una infección primaria. La transmisión es de 30 a 50% cuando la infección primaria
ocurre cerca del momento del parto. La mayor transmisión (85%) ocurre durante el
parto. Sin embargo, también puede ocurrir transmisión intrauterina (5-8%) y post-natal
(8-10%). Los factores que inciden en la transmisión son: infección primaria mucho
mayor eficiencia que infección recurrente, parejas discordantes, títulos de anticuerpos
maternos y procedimientos obstétricos invasores, (los que están absolutamente
contraindicados).

Las manifestaciones en la mujer embarazada son principalmente bajo la forma de


herpes genital localizado, muy raramente ocurre diseminación cutánea y visceral,
situación de elevada mortalidad (50%). La infección en el niño, si ocurre en las
primeras 20 semanas del embarazo, puede provocar aborto en 25%, malformaciones
cerebrales, cicatrices, corioretinitis, RCIU. Si ocurre después de las 20 semanas,
puede causar parto prematuro, RCIU, o herpes neonatal. Esta condición clínica tiene
tres formas de presentación, las dos primeras de elevada mortalidad y secuelas:
herpes diseminado y encefalitis herpética o infección localizada en piel, ojo y boca.

Pass R, Weber T, Whitley RJ. Herpesvirus infections in pregnancy.


Recommendations from the International Herpes Management Forum.
Management Strategies Workshop and 7th Annual Meeting.

Whitley R J. Varicella - Zoster virus. Mandell, Douglas and Bennett's


Principles and Practice of Infectious Diseases. Mandell G, Bennett J, Dolin
R, editors. Fifth edition, 2000 Churchill Livingstone, Philadelphia, pp: 1586-
98.

32.- Femenino de 26 años, que presenta una tumoración de 2 cm de diámetro en el


cuadrante ínfero-externo de la mama izquierda, indolora, de consistencia firme,
superficie lisa, forma ovoidea, móvil y bien delimitada del parénquima vecino, sin
antecedentes de derrame por el pezón, sin “piel de naranja” ni retracción del
pezón,. El diagnóstico más probable es:

a) Fibroadenoma.
b) Carcinoma.
c) Ectasia de los conductos mamarios.
d) Quiste solitario.

FIBROADENOMA MAMARIO
Tumor benigno más frecuente en las mujeres entre los 20 y 35 años.
ETIOLOGIA
Existen múltiples teorías siendo la más aceptada la hormonal, generalmente son
únicos, solo el
20% son múltiples o bilaterales. De tamaño variable hasta de 10 cm. Ocupa el 13.6%
de la patología mamaria benigna.
CUADRO CLÍNICO
Lesión nodular de consistencia dura, de larga evolución y no dolorosa. Normalmente
llegan a los 3 cm. De diámetro. Durante la fase tardía del ciclo menstrual el tumor
suele presentar un leve aumento de tamaño. Durante la menopausia presentan
regresión hasta la calcificación (signo de palomitas de maíz).
DIAGNOSTICO
Es clínico, se presenta como un tumor bien delimitado, desplazable, no adherido a piel
ni a planos profundos, lisos o multilobulados en ocasiones. Se localiza frecuentemente
en cuadrantes externos.
EXAMENES DIAGNOSTICOS
ULTRASONIDO MAMARIO .- Identifica un nódulo sólido, bien delimitado de bordes
regulares .
TRATAMIENTO.-
Conservador con vigilancia estrecha dependiendo del tamaño y en caso de ser
necesario exéresis del nódulo para estudio histopatológico
hospitalgeneral.salud.gob.mx/
BIBLIOGRAFIA
1. Sánchez BC. Tratado de Enfermedades de la glándula mamaria. Ed. Manual
Moderno. Cap. 13- 15.
2.- De Vita V. Cancer of the Breast. In Cancer: Principles and Practice of Oncology:
Fifth Ed. Philadelphia: Lippincott-Raven, Chapter 36; pp: 1521-1616.
3.-Consenso Nacional Acerca del Tratamiento de Cáncer de Mama. En Tumores de
mama: Diagnóstico y Tratamiento. 2ª Ed. McGraw-Hill Interamericana; pp: 119-126.
4.-Eberlein T. Current management of carcinoma of the breast. Ann Surgery 1994;
220: 121-136.
5. Encyclopedie Medico. Chirurgicale Praxis Médica, Editions Techiques de Mexico,
tomo 5, año 2005.

33.- Femenino de 23 años, G1 en trabajo de parto prematuro con embarazo de 30


semanas de gestación. A pesar del uso de agentes tocolíticos, estos no han dado
resultado. Se puede inducir la maduración pulmonar del producto por medio de:

a) Betametasona

b) Sulfato de magnesio

c) Hidroxiprogesterona

d) Clorprocaína

La utilización de betametasona como inductor de madurez pulmonar fetal (IMPF)


disminuye la morbilidad neonatal relacionada con prematurez pero su efecto
diabetógeno materno ha sido poco estudiado.

La revisión Cochrane de un ciclo único de corticosteroides se actualizó en 2006. En


esta actualización se incluyeron 21 estudios con un total de 3885 mujeres y 4269
lactantes.

En la revisión se descubrió que la administración de determinados corticosteroides a


mujeres con riesgo de tener un parto prematuro reduce considerable los riesgos de
complicaciones relacionadas con la prematurez como muerte fetal y neonatal
combinada, síndrome de dificultad respiratoria, hemorragia cerebroventricular,
enterocolitis necrotizante, infecciones sistémicas y retraso en el desarrollo durante la
niñez. Los beneficios estaban presentes cuando el tratamiento se iniciaba entre las 26
y las 35 semanas de gestación y en los niños que nacían entre 1 y 7 días después de
haber comenzado el tratamiento; también se observaron beneficios en los subgrupos
de mujeres con rotura prematura de membranas y trastornos hipertensivos. La muerte
fetal y neonatal combinada se redujo incluso en neonatos que nacieron a menos de las
24 horas de haber administrado la primera dosis.
No se demostraron beneficios cuando el tratamiento comenzó antes de las 26
semanas de gestación, tampoco se observaron beneficios en los recién nacidos antes
de las 26 semanas de gestación ni en los que nacieron después de 7 días o más de la
administración del tratamiento.

En el caso de los neonatos que nacieron después de las 36 semanas hubo una
tendencia a aumentar la muerte fetal y neonatal combinada.

Se observó una reducción en el peso al nacer en los neonatos que nacieron entre los
días 1 y 7, al igual que en los que nacieron más de 7 días después del primer
tratamiento.

Un estudio que reclutó mujeres con preeclampsia severa sugirió que las mujeres
tratadas tenían un mayor riesgo de sufrir diabetes gestacional.

La evidencia epidemiológica y en animales sugiere que puede haber efectos adversos


a largo plazo por la exposición prenatal a los corticosteroides, entre ellos la alteración
de la tolerancia a la glucosa y la hipertensión. Los estudios en animales también han
sugerido que afecta el crecimiento del cerebro.

Prevención, diagnóstico y tratamiento del Parto Pretérmino. Guía de Referencia


Rápida: Guía de Práctica Clínica. México, CENETEC; Disponible en:
http://www.cenetec.salud.gob.mx/contenidos/gpc/catalogoMaestroGPC.html

34.- Se trata de paciente femenino de 24 años la cual presenta un nódulo mamario


palpable de aparición brusca. La ecografía revela un nódulo anecogénico, de limites
muy precisos, morfología regular y refuerzo posterior, único de 3.5 cms. de diámetro.
El diagnóstico más probable es:

a) Cáncer.
b) Displasia fibrosa.
c) Fibroadenoma.
d) Quiste

Quistes. Los quistes mamarios son fáciles de detectar con la ecosonografía. Pueden
ser lesiones únicas o múltiples que se observan como imágenes redondeadas,
anecogénicas, de paredes delgadas, contornos bien definidos, con importante
reforzamiento acústico posterior y sombras laterales delgadas. Pueden presentar
septos intraquísticos y, en ocasiones, se pueden observar ecos internos que sugieren
detritus celulares o proceso inflamatorio. Se debe descartar la presencia de lesiones
intraquísticas o la coexistencia de otras alteraciones benignas o malignas. En caso de
ser sintomáticos, el tratamiento adecuado es la punción y aspiración de la lesión con
aguja guiada por palpación o ecosonografía de acuerdo con ell tamaño, profundidad y
características del contenido. El uso del ultrasonido garantiza el vaciamiento completo.
REFERENCIAS
Barth V, Prechtel K. Mama normal. En: Barth V, Prechtel K, editores. Atlas de
patología de la glándula mamaria. Ultimaa dcion
Bush H, McCredie A. Carcinoma of the breast during pregnancy and lactation. In: Allen
HH, Nisker JA. Cancer in pregnancy. New York: Futura Publishing Co. Inc., 6.
Byrd BF, Bayer DS, Robertson JC, Stephenson JE Jr.

35.-Se trata de mujer de 36 años, acude al servicio de consulta externa externa con
reporte de papanicolaou que incica un NIC I, la especuloscopía se observa cérvix con
ectropión periorificiario.
El agente etiológico más probable causante de esta infección es:

a) neisseria gonorreae.
b) clamidya trachomatis.
c) virus del papiloma humano.
d) treponema pallidum.
9.5.2 Las pacientes a quienes se les realizó citología cervical, cuyo resultado es
LEIBG (infección por VPH, displasia leve o NIC 1); LEIAG (displasia moderada y grave
o NIC 2 y 3) o cáncer deben enviarse a una clínica de colposcopía, para realizar
estudio colposcópico.
9.5.3 Si el resultado de la citología es LEIBG, la colposcopía es satisfactoria y sin
evidencia de LEIBG, se realizará control citológico en un año (Apéndice Normativo A)
9.5.4 Si la citología es de LEIBG, la colposcopía es satisfactoria y existe evidencia de
lesión, se debe tomar una biopsia dirigida.
9.5.4.1 Si la biopsia dirigida es negativa, se realizará nueva colposcopía para verificar
el diagnóstico y en caso necesario, tomar nueva biopsia dirigida y revalorar.
9.5.4.2 Si la biopsia dirigida es reportada como LEIBG se podrá dar tratamiento
conservador: criocirugía, electrocirugía o laserterapia (sólo si cumple con las
condiciones referidas en el Apéndice 1) o se podrá mantener a la paciente en
vigilancia en la clínica de colposcopía, con colposcopía y estudio citológico cada seis
meses, durante 24 meses.
Jueves 31 de mayo de 2007 DIARIO OFICIAL (Primera Sección)
9.5.4.3 Si la biopsia dirigida es reportada como LEIAG (Lesión Intraepitelial Escamosa
de Alto Grado) se realizará tratamiento conservador (electrocirugía o laserterapia). En
las mujeres posmenopáusicas, dependiendo de las condiciones anatómicas del cérvix,
se realizará tratamiento conservador en la clínica de colposcopía o tratamiento
quirúrgico (histerectomía extrafascial) en el servicio que corresponda.
9.5.4.4 Si la biopsia dirigida reporta cáncer microinvasor o invasor, la paciente se
transferirá a un Servicio o Centro Oncológico para su tratamiento correspondiente.
9.5.4.5 Si la citología reporta LEIBG y la colposcopía es no satisfactoria, se tomará
cepillado endocervical (Apéndice Normativo A)
9.6 En caso de colposcopía no satisfactoria, negativa a LEIBG y con cepillado
endocervical negativo, se continuará su control en la clínica de colposcopía en seis
meses, con colposcopía y citología.
9.6.1.1 Si el cepillado endocervical reporta LEIBG se tratará a la paciente como
LEIAG, con métodos conservadores escisionales.

Jueves 31 de mayo de 2007 DIARIO OFICIAL (Primera Sección)


Modificación a la Norma Oficial Mexicana NOM-014-SSA2-1994, Para la
prevención, detección, diagnóstico, tratamiento, control y vigilancia
epidemiológica del cáncer cérvico uterino.
Al margen un sello con el Escudo Nacional, que dice: Estados Unidos Mexicanos.-
Secretaría de Salud.
MODIFICACION A LA NORMA OFICIAL MEXICANA NOM-014-SSA2-1994, PARA LA
PREVENCION,
DETECCION, DIAGNOSTICO, TRATAMIENTO, CONTROL Y VIGILANCIA
EPIDEMIOLOGICA DEL CANCER CERVICO UTERINO.

El agente etiológico del cáncer de cuello uterino es el “papiloma virus humano” (hpv).

Existen lesiones precursoras del cáncer cervical, son las llamadas lesiones
intraepiteliales de cuello uterino (también conocidas como sil: squamous
intraepithelial lesion) son lesiones que no atravesaron la membrana basal del epitelio y
que por lo tanto no pueden invadir ni diseminarse por el resto del cuerpo, como sí lo
hace un cáncer invasor. anteriormente a las lesiones intraepiteliales se las llamaba
neoplasia intraepitelial cervical (también conocidas como cin: cervical intraepithelial
neoplasia), y anteriormente se las llamaba displasias de cuello uterino.
36.- Femenino de 40 años de edad G.3 P.1 C 2, la cual es diagnosticada por
miomatosis uterina de pequeños y medianos elementos actualmente sintomática, con
Hb 10.1 gr sus antecedentes refieren cirugías pélvicas previas, el tratamiento de
elección en esta paciente es:

a) Histerectomía total abdominal.

b) Progesterona.

c) Observación

d) Análogos de GnRH.

CUADRO CLINICO

La miomatosis uterina muestra manifestaciones clínicas en menos del 50%, de estas


las más frecuentes son:

1. Hemorragia uterina anormal.


2. Dolor.
3. Distensión abdominal.
4. Compresión genitouterina.
5. Compresión gastrointestinal.
6. Compresión pélvica.

10. DIAGNOSTICO

El diagnóstico se realiza a través de imagenología:

1-Ecografía.
2-TAC
3-Rayos X
4-Histeroscopia.

11. TRATAMIENTO

La miomatosis uterina debe ser tratada cuando produzca cualquiera de las


manifestaciones clínicas anotadas, toda paciente que se programe para histerectomía
debe tener previamente legrado biopsia

Lo podemos dividir en Conservador o Radical.

1. CONSERVADOR:
Este tratamiento se puede instaurar en pacientes con deseo de preservar el útero.
Igualmente se puede subdividir:
-Expectante
-Quirúrgico: miomectomía
-Medico

Tratamiento expectante: Esta indicado en pacientes, cuyos síntomas son leves y no


deseen o tengan alguna contraindicación médica para tratamiento quirúrgico. En ellas
se recomienda controles clínicos y ecográficos cada 6 meses a 1 año.

Tratamiento médico.

AINES

Análogos GnRH:

Progestágenos:
Andrógenos.

Antiandrógenos

Tratamiento quirúrgico:

MIOMECTOMIA:

1. Criterios del ACOG en pacientes infecundas.

Procedimientos:

 Vía endoscópica:
Laparoscopia: Miomas subserosos sesiles o pediculados < 5cm.
Histeroscopia: miomas submucosos

 Vía laparotomía
Aquellos miomas que se salgan de las características anteriores.

Indicaciones:

 Hemorragia anormal.
 Perdida reproductiva.
 Infecundidad.
 Dolor.

El tratamiento previo con análogos está indicado cuando se desee disminuir el tamaño
del mioma para prevenir sangrado quirúrgico.

Contraindicaciones:

 Embarazo.
 Cáncer endometrial.
 Infecciones.
 Dificultad técnica.
2. Criterios del ACOG para Miomectomía en pacientes que desean conservar el útero.

Procedimiento:

 Vía endoscópica.
 Vía abdominal.
 Vía vaginal.

Indicación:

a. Presencia de uno o dos Leiomiomas asintomáticos de tamaño tal que se pueden


palpar por vía abdominal y constituyen una preocupación para la paciente.

b. Pacientes ovulatorias con miomas como posible causa de hemorragia uterina


excesiva, demostrada por cualquiera de las siguientes circunstancias:
 Hemorragia profusa: de duración mayor de 8 días.
 Anemia por pérdida sanguínea aguda o crónica.

2. RADICAL:

HISTERECTOMIA

Para pacientes post menopáusicas, con paridad satisfecha o sin deseo de preservar el
útero.

Criterios del ACOG para Histerectomía por miomas.

1. Presencia de 1, 2, o 3 Miomas asintomáticos de tamaño tal que son palpables por


vía abdominal y preocupan a la paciente.

2. Hemorragia uterina excesiva.

 Duración mayor de 8 días.


 Anemia por pérdida sanguínea aguda o crónica.

1. Molestias pélvicas producidas por los miomas: signos compresivos.

Contraindicaciones:

1. Deseo de conservar la fecundidad.


2. Miomas asintomáticos.
3. Contraindicación médica o dificultades técnicas para la cirugía.

Guarnaccia M. and Rein M. Traditional Surgical Approaches to Uterine Fibroids


Abdominal. Myomectomy and Hysterectomy. Clinical Obstetrics and Gynecology
2001. 44.2. 385-400
- Milad. M and Sankpal R. Laparoscopic Approaches to Uterine Leiomyomas.
Clinical Obstetrics and Gynecology. 2001. 44-2. 401-411
- Carlson K. et al. Indications for Hysterectomy. N. Engl. J. Med. 1993. 328(12) 56-
37.- Se trata de paciente femenino con diagnóstico de amenorrea, acude con
resultados de laboratorio los que reportan los siguientes niveles hormonales: GnRH
elevada, FSH y LH elevadas, hormonas ováricas (estrógenos y progesterona) bajas.
El defecto está a nivel de:

a) Hipotálamo.

b) Hipófisis.

c) Ovario.

d) Endometrio.

Evaluación de la paciente con amenorrea secundaria

La mayoría de las pacientes con AS que no están embarazadas o no entraron en el


climaterio tienen una alteración en algún nivel de la cascada reguladora del ciclo
menstrual femenino. A continuación, esquematizamos la evaluación de la AS en
pasos. Cada médico deberá adaptar los tiempos de la evaluación a la situación
particular de cada paciente. No obstante, recomendamos no saltear ninguno de estos
pasos ya que un diagnóstico preciso permitirá realizar un tratamiento racional y dar un
pronóstico respecto de la futura función menstrual y de la fertilidad.

Primer paso (descartar el embarazo)

Su prueba se basa en el dosaje cuali o cuantitativo de la subunidad beta de la


gonadotrofina coriónica humana. Si se certifica la ausencia de embarazo podrá
avanzarse al siguiente paso, pero teniendo en cuenta siempre que si la probabilidad
de embarazo es muy alta se deberá repetir la prueba nuevamente. Se recomienda no
obviar esta prueba aunque la paciente asegure que no ha mantenido relaciones
sexuales.

Segundo paso (prueba de progesterona)

Debe realizarse sólo si el test de embarazo es negativo y consiste en dar


medroxiprogesterona entre 30 a 50mg por vía oral (un comprimido de 10mg durante 5
días) o progesterona oleosa 100 a 200mg por vía intramuscular en una sola dosis. El
resultado de esta prueba provee información acerca de si se produjeron estrógenos.
La prueba se considera positiva si se produce un sangrado luego de 2 a 14 días de la
suspensión de la progesterona. La respuesta positiva indica que existe integridad
anatómica del aparato genital femenino y que el ovario produce estrógenos. En estos
casos, la AS se debe a que no ha habido ovulación. La causa más frecuente de AS
con prueba de progesterona positiva es la disfunción hipotalámica leve. En este caso,
no se dispara el pico de LH necesario para producir la ovulación, no se ovula (ciclo
anovulatorio), no hay cuerpo lúteo y no hay producción de progesterona. El sistema
reproductor queda en un estado folicular, con grados variables de desarrollo folicular,
acompañados de concentraciones cambiantes de estradiol. Puede producirse
hemorragia en ausencia de ovulación dado que el endometrio proliferativo sufrirá
ocasionalmente un desprendimiento parcial en respuesta a estos niveles cambiantes
de estradiol. Estos sangrados luego de un ciclo anovulatorio se denominan sangrados
uterinos disfuncionales (SUD). Como dijimos, ejemplos frecuentes de situaciones de
anovulación son el estrés, la pérdida brusca de peso y el ejercicio intenso. Otras
causas menos frecuentes son la hiperprolactinemia, el hipotiroidismo y el sindrome del
ovario poliquístico. Otras veces no hay una causa clara (idiopática) de este trastorno.

La prueba se considera negativa si NO se produce un sangrado luego de 2 a 14 días


de la suspensión de la progesterona, lo que ocurre cuando los niveles de estrógenos
son bajos (menores a 40 pg/ml). La causa más frecuente de prueba negativa es la
insuficiencia gonadal de la menopausia.

Tercer paso

Para avanzar al tercer paso es preciso distinguir si la prueba de progesterona fue


positiva o negativa.

Pacientes con prueba de progesterona positiva

Una prueba de progesterona positiva hace diagnóstico de ciclos anovulatorios. Según


los antecedentes, el examen clínico y la probabilidad previa el médico podrá orientarse
hacia cuál es el diagnóstico. Hay situaciones de estrés, crisis vitales, cambios de peso
brusco o ejercicio extremo que ocasionan un trastorno del ciclo aislado. En estos
casos, la prueba de progesterona es diagnóstica y terapéutica. Es decir, la paciente no
requiere estudios posteriores. Es conveniente reasegurarla y explicarle que su periodo
se normalizará una vez solucionado el problema. En una paciente con trastornos
anovulatorios frecuentes, que presenta obesidad, acné e hirsutismo, existe alta
sospecha de síndrome de ovario poliquístico (SOP). En este caso, debemos solicitar
una ecografía pelviana y un dosaje de LH/FSH o de hormonas masculinas:
testosterona libre (To), dehidroepiandrosterona sulfato (DHEA-S) y 17OH
progesterona.

Si la paciente tiene trastornos del ciclo y galactorrea, se debe solicitar un dosaje de


prolactina (PRL). El valor normal de PRL para la mujer no embarazada es de 20 a
25ng/ml. Cuando el valor de PRL es menor de 100ng/ml, generalmente es de causa
idiopática o farmacológica; si está entre 100 y 200ng/ml, puede ser por causa
farmacológica o por enfermedad hipotalámica; si es mayor de 200ng/ml, la causa más
probable es el adenoma hipofisario. Cuando el valor de la PRL es mayor de 100ng/ml,
se debe solicitar un estudio por imágenes que puede ser una tomografía computada
de cerebro con contraste o una resonancia magnética nuclear con gadolinio. Si se
observa una imagen menor de 10mm, se trata de un microadenoma de hipófisis y si es
mayor de 10mm, se hace diagnóstico de macroadenoma. Si la paciente consulta por
trastornos del ciclo frecuentes y no presenta galactorrea o signos o síntomas de
hipotiroidismo, igualmente deberá solicitarse un dosaje de tirotrofina (TSH) y de
prolactina (PRL) séricas.

Pacientes con prueba de progesterona negativa


Deberá dosarse el nivel sérico de FSH cuyo valor refleja los niveles de estrógenos
circulantes (por el fenómeno de retroalimentación negativa). Si la FSH es mayor de 40
UI/ml, los niveles de estrógenos son bajos. En este caso, se asume que existe una
insuficiencia ovárica. El valor de la FSH puede hacer el diagnóstico diferencial entre la
falla ovárica (castración temprana, menopausia precoz o menopausia normal) y la
disfunción hipotálamo-hipofisaria severa. En la primera, la ausencia de
retroalimentación negativa estimula la liberación de las gonadotrofinas y, en
consecuencia, éstas se encuentran elevadas en la sangre. En la segunda, los valores
de gonadotrofinas son bajos debido a un defecto en su producción. El dosaje de FSH
es más sensible que el de LH para este propósito, por lo tanto, si la prueba de
progesterona es negativa y la FSH es baja, lo más probable es que la causa sea una
disfunción hipotálamo-hipofisaria severa. En este caso debería solicitarse un estudio
de diagnóstico por imagen del cerebro (RMN con gadolinio o TAC con contraste). En el
cuadro 4 se muestran los valores normales de las hormonas mencionadas.

Cuadro 4: valores hormonales normales


PRL (prolactina): 20 a 25ng/ml en mujeres no embarazadas.
TSH: de 0.5 a 5mUI/ml.
FSH: 5 a 30mUI/ml.
LH: 5 a 20mUI/ml (en el pico ovulatorio este valor se debe multiplicar por dos o tres).
Falla ovárica: FSH mayor de 40mUI/ml.
Disfunción hipotálamo hipofisaria: FSH menor a 5mUI/ml y LH menor a 5mUI/ml.

En general, una paciente con disfunción hipotalámica con gonadotrofinas bajas está
expresando una gravedad mayor que la disfunción hipotalámica leve con prueba de
progesterona positiva.
En los casos en los que la prueba de progesterona es negativa, la FSH es normal, la
paciente no está en una edad cercana a la menopausia y existe alta sospecha de que
exista una enfermedad uterina, se recomienda realizar una “prueba de estrógenos y
progesterona”. Esta prueba sirve para conocer si el efector (el útero) responde a los
estímulos hormonales. Consiste en administrar estrógenos (1.25mg por día durante 20
días) más progesterona (10mg por día durante los últimos 5 días en que se
administran los estrógenos). Si no hay sangrado, se debe repetir la prueba y, si
nuevamente no hay sangrado, la prueba se considera negativa. Esto indica que hay
una falla en el efector, o sea, en el útero.

En este caso la paciente debe ser derivada al ginecólogo. Las causas más probables
de falla uterina son la endometritis por abortos o partos sépticos y las sinequias por
curetajes vigorosos. Cuando falla el efector, la ecografía transvaginal puede ser
normal. Por eso debe realizarse una prueba de estrógenos y progesterona para
evaluar directamente la funcionalidad del útero. La prueba se considera positiva si hay
sangrado (aunque sean sólo gotitas). Esto significa que el útero está indemne para
responder a estímulos hormonales y el problema es de origen hipotálamo- hipofisario

1. Jonathan R, Pletcher, Gail B. Slap. Menstrual Disorders Amenorrhea. Pediatric


Clinics of North America. June 1999; 46: Issue 3.
2. Bryan McIver, Susan A Romanski; Todd B Nippoldt. Evaluation and
Management of Amenorrhea. Mayo Clinic Proceedings Dec 1997; 72: 1161-
1169.
3. Tarannun Master Hunter, Diana L.Heiman. Amenorrhea: Evaluation and
Treatment. Am Fam Physician 2006; 73: 1374-1382.
38.- Se trata de femenino de 33 años la cual ingresa a la sala de urgencias, inicia con
convulsiones por presentar preclampsia, se decide iniciar tratamiento, el fármaco
elección en ésta patología es:

a) Diacepam.
b) Fenitoína.
c) Donadores de óxido nítrico.
d) Sulfato de magnesio.

Manejo de la Preeclampsia

1. Manejo ambulatorio: HTA sin proteinuria significativa, se recomienda el reposo en


cama. Monitoreo de TA, peso, presencia de proteínas en orina. Ecografías periódicas
para ver el feto y evaluar posibles retardo de crecimiento.
2. Manejo hospitalario: para mujeres con HTA inducida por el embarazo y 2+ o más o
proteinuria significativa y en quienes falló el manejo ambulatorio.
3. Laboratorio y evaluación del peso: debe realizarse diariamente. Evaluación de la
dinámica fetal. Monitoreo de síntomas como cefalea, alteraciones visuales y dolor
epigástrico.
4. El parto es el tratamiento de elección: el cual debe realizarse cuando el feto está
maduro pero puede realizarse en forma temprana si la salud de la madre está en
peligro o si hay evidencia de distress fetal. El parto está indicado cuando la paciente
cumple con los criterios de preeclampsia severa. Betametasona 12.5 mg IM dos veces
por día puede estimular la maduración de los pulmones fetales.
5. Terapia antihipertensiva: está indicada sólo si la TA es persistentemente > 160/110
, es importante disminuir la TA hasta una diastólica de 90 a 100 mmHg porque la
presión normal podría resultar en hipoperfusión de la placenta. Los diuréticos nunca
están indicados, estas pacientes ya son hipovolémicas. Los IECA no deben ser
usados durante el embarazo. Las medicaciones de largo plazo, incluyen alfa
metildopa, atenolol y labetalol.
6. Terapia anticonvulsivante:
A- Profilaxis de las convulsiones: está indicada en todas las pacientes pre-
eclámpticas durante el trabajo de parto y el parto y por un mínimo de 24 hs luego del
mismo. Algunos mantienen la terapia con magnesio hasta que comienza la diuresis. El
Sulfato de Magnesio es la droga de elección. La dosis profiláctica es de 4 a 6 g de
sulfato de magnesio IV y continúa con 2 g c/ hora.
B- Tratamiento de las convulsiones: Sulfato de Magnesio 1 g/min IV hasta
controlar las convulsiones hasta un máximo de 4 a 6 g. El nivel terapéutico es de 4
meq/l. Toxicidad del magnesio: ausencia de reflejo patelar, debilidad muscular,
parálisis respiratoria y depresión cardíaca, 10 ml al 10 % de gluconato de calcio puede
ser administrada IV. La terapia con sulfato de magnesio continúa por lo menos 24
horas en el post parto, la terapia puede detenerse si la excreción urinaria es > 200
ml/h por cuatro horas consecutivas.
C- Prevención: 81 mg de aspirina diarios pueden ser administrados luego del
primer trimestre en mujeres con hipertensión crónica o historia previa de preeclampsia,
sin embargo la eficacia de esta indicación ha sido cuestionada.

Myers JE, Baker PN. Hupertensive diseases and eclampsia. Curr Opin Obstet
Gynecol 2002; 14: 119-125
2. Tierney, McPhee, Papadakis. Diagnóstico clínico y tratamiento 2003. 38ª ed,
México, Manual Moderno, 2003: 770-773
3. Wilson MI, Goodwin TM, Pan VI, Ingles SA. Molecular epidemiology of
preeclampsia. Obstet and Gynecol Survey 2003; 58(1):39-66
4. Burrow GM. Complicaciones médicas durante el embarazo. 4ª ed, México, McGraw-
Hill panamericana: 1996: 1-25
5. Guyton AC, Hall JE. Embarazo y lactancia en: Tratado de fisiología médica, 10ª ed,
México, McGraw-Hill Interamericana 2001: 1135-45
6. Vaticon D. Fisiología de la fecundación, embarazo, parto y lactancia, en:
Tresguerres JAF. Fisiología Humana. México, Interamericana McGraw-Hill, 1992:
1086-1109
7. Pridjian G, Puschett JB. Preeclampisa. Part 1: Clinical and Pathophysiologic
Considerations. Obstet and Gynecol Survey 2002; 57 (9): 598-618
8. Pridjian G, Puschett JB. Preeclampisa. Part I1: Experimental and Genetic
Considerations. Obstet and Gynecol Survey 2002; 57 (9): 619-40
9. IMSS. Embarazo de alto riesgo. Guía diagnóstica terapéutica. Rev Med IMSS 1998;
36(1):45-60

39.- Acude a consulta una mujer de 26 años, cursando su 14ª semana de gestación,
por tenesmo vesical, disuria y escalofrío. Por su estado actual, el antimicrobiano de
elección para esta paciente es:

a) Ampicilina
b) TMP/SMZ
c) Tetraciclina
d) Levofloxacina

El Metronidazol no ha mostrado efectos tóxicos en humanos, pero es teratogénico en


modelo animal. Las tetraciclinas ocasionan coloración anormal de los dientes,
hepatotoxicidad y alteración en el desarrollo de huesos. Las sulfas podrían tener un
efecto deletéreo en el primer trimestre dada su actividad como antimetabolitos, y en
los últimos meses pueden favorecer kernicterus en el recién nacido si es que tiene
alteraciones metabólicas que favorezcan anemia hemolítica. Las quinolonas se han
asociado a malformaciones óseas en modelos animales, y se recomienda evitarlas si
existen mejores opciones.

Kasper DL, Braunwald E, Fauci AS, Hauser SL, Longo DL, Jameson JL. Harrison´s
Principles of Internal Medicine. McGraw Hill. 16 Ed. 789-806 pp.

40.- Which of the following drugs combination would be most appropriate in the patient
treatment of acute pelvic inflammatory disease?

a) Ampicillin / cefoxitin
b) Tetracycline / gentamicin
c) Ceftriaxone / Doxycycline
d) Ampicillin / Amikacin
E.P.I.
Regímenes de tratamiento CDC 2002

• Parenteral.
– Cefotetan 2gr iv/12h ó Cefoxitina 2gr iv/6h + Doxiciclina 100 mgs iv/8h.
– Clindamicina 900 mg iv/8h + Gentamicina iv/im (2mg/kg de carga, luego
1.5 mg/kg/8h.
• Alternativas.
– Ofloxacina 400 mg iv/12h ó Levofloxacino 500 mg iv diario con o sin
Metronidazol 500 mg iv/12 h.

• Oral.
– Ofloxacina 400 mg vo diario x 14 días ó Levofloxacina 500 mg vo diario
x 14 días con o sin Metronidazol 500 mg vo diario por 14 días.
– Ceftriaxona 250 mg im x 1 dosis ó Cefoxitina 2 gr im x 1 dosis y
Probenecid 1 gr vo x 1 dosis u otra cefalosporina de 3ª gen im +
Doxiciclina 100 mgs vo diario or 14 días con o sin Metronidazol 500
mgs vo diario x 14 días.

41.- Se trata de paciente femenino de 67 años que acude a consulta externa, refiere
intenso prurito vulvar y sensación quemante, a la especuloscopía el introito vaginal se
encuentra estenótico. De los siguientes, el tratamiento más apropiado en este caso es:

a) 5-fluoracilo
b) Testosterona tópica
c) Corticoesteroides fluorados
d) Estrógeno tópico

Vulvovaginitis atrófica

El hipoestrogenismo conduce a atrofia de la vagina y el vestíbulo vulvar, que los hace


fácilmente irritables y susceptibles a infecciones secundarias. Las pacientes refieren
sensación de quemadura, prurito, disuria, hipersensibilidad y dispareunia. Puede
encontrarse al examen físico atrofia, fisuras superficiales, y un flujo vaginal acuoso1.
Hay disminución del tamaño del introito2, pérdida de la rugosidad y la vagina toma una
apariencia lisa y brillante.

Los hallazgos histológicos revelan un epitelio vaginal delgado, disminución de los


lechos capilares, y la citología muestra, a medida que la atrofia progresa, aumento de
las células basales y disminución o ausencia de las células superficiales2.

Se aconseja evitar el uso de jabones y demás irritantes de la piel. Se pueden utilizar


lubricantes simultáneamente con los estrógenos o como terapia única, si hay alguna
contraindicación a las hormonas.
El tratamiento con estrógenos por vía sistémica o transvaginal mejora y restaura los
signos y síntomas, y una a dos semanas después de iniciar el tratamiento los cambios
de atrofia empiezan a mejorar rápidamente, se reduce el pH y se induce maduración
vaginal y de la mucosa uretral, reduciendo la frecuencia de las infecciones urinarias3.
La dosis y vía de administración debe ser debidamente individualizada4.
Contraindicaciones al tratamiento con estrógenos, incluyen: la presencia de tumores
estrógenosensibles, falla hepática terminal y antecedentes de tromboembolización
relacionada con ellos.

Menopausia y Piel. Parte II: Manifestaciones clínicas


dermatológicas durante la menopausiaMARÍA ISABEL BARONA C. ultima ed.

42.- Mujer de 26 años, es atendida en consulta en la clínica de displasias por


papanicolau con lesión NIC I. Antecedentes: menarca 14 años, ritmo 30x5
eumenorreica, inicio de vida sexual a los 15 años, 2 parejas sexuales, método de
planificación familiar oclusión tubaria bilateral, gestas 3 partos 3, cérvix con lesión
acetoblanca con extensión lineal de 2 cm. Para confirmar el diagnóstico se debe
realizar:

a) Crioterapia de lesión.
b) Biopsia de la lesión.
c) Captura de híbridos.
d) Repetir colposcopía
9.5.2 Las pacientes a quienes se les realizó citología cervical, cuyo resultado es
LEIBG (infección por VPH, displasia leve o NIC 1); LEIAG (displasia moderada y grave
o NIC 2 y 3) o cáncer deben enviarse a una clínica de colposcopía, para realizar
estudio colposcópico.
9.5.3 Si el resultado de la citología es LEIBG, la colposcopía es satisfactoria y sin
evidencia de LEIBG, se realizará control citológico en un año (Apéndice Normativo A)
9.5.4 Si la citología es de LEIBG, la colposcopía es satisfactoria y existe
evidencia de lesión, se debe tomar una biopsia dirigida.
9.5.4.1 Si la biopsia dirigida es negativa, se realizará nueva colposcopía para verificar
el diagnóstico y en caso necesario, tomar nueva biopsia dirigida y revalorar.
9.5.4.2 Si la biopsia dirigida es reportada como LEIBG se podrá dar tratamiento
conservador: criocirugía, electrocirugía o laserterapia (sólo si cumple con las
condiciones referidas en el Apéndice 1) o se podrá mantener a la paciente en
vigilancia en la clínica de colposcopía, con colposcopía y estudio citológico cada seis
meses, durante 24 meses.
Jueves 31 de mayo de 2007 DIARIO OFICIAL (Primera Sección)
9.5.4.3 Si la biopsia dirigida es reportada como LEIAG (Lesión Intraepitelial Escamosa
de Alto Grado) se realizará tratamiento conservador (electrocirugía o laserterapia). En
las mujeres posmenopáusicas, dependiendo de las condiciones anatómicas del cérvix,
se realizará tratamiento conservador en la clínica de colposcopía o tratamiento
quirúrgico (histerectomía extrafascial) en el servicio que corresponda.
9.5.4.4 Si la biopsia dirigida reporta cáncer microinvasor o invasor, la paciente se
transferirá a un Servicio o Centro Oncológico para su tratamiento correspondiente.
9.5.4.5 Si la citología reporta LEIBG y la colposcopía es no satisfactoria, se tomará
cepillado endocervical (Apéndice Normativo A)
9.6 En caso de colposcopía no satisfactoria, negativa a LEIBG y con cepillado
endocervical negativo, se continuará su control en la clínica de colposcopía en seis
meses, con colposcopía y citología.
9.6.1.1 Si el cepillado endocervical reporta LEIBG se tratará a la paciente como
LEIAG, con métodos conservadores escisionales.
Jueves 31 de mayo de 2007 DIARIO OFICIAL (Primera Sección)
Modificación a la Norma Oficial Mexicana NOM-014-SSA2-1994, Para la
prevención, detección, diagnóstico, tratamiento, control y vigilancia
epidemiológica del cáncer cérvico uterino.
Al margen un sello con el Escudo Nacional, que dice: Estados Unidos Mexicanos.-
Secretaría de Salud.
MODIFICACION A LA NORMA OFICIAL MEXICANA NOM-014-SSA2-1994, PARA LA
PREVENCION,
DETECCION, DIAGNOSTICO, TRATAMIENTO, CONTROL Y VIGILANCIA
EPIDEMIOLOGICA DEL CANCER CERVICO UTERINO.

7.3 El resultado del estudio citológico es descriptivo y debe ser informado de la


siguiente manera:

a.- Negativo a cáncer.

b.- Negativo con proceso inflamatorio.

c.- Displasia leve (NIC 1).

d.- Displasia moderada (NIC 2).

e.- Displasia grave (NIC 3).

f.- Cáncer del cuello del útero in situ (NIC 3).

g.- Cáncer microinvasor e invasor.

h.- Adenocarcinoma.

i.- Maligno no especificado.

43.- Se trata de paciente de 34 años que cursa con 39 SDG; a la exploración física
reflejos patelares hiperactivos, inquieta, se reportan cifras de TA 145/95, se realiza
laboratorio que reporta proteinuria 2+. El diagnóstico más probable es:

a) Glomerulonefritis aguda
b) Hipertensión esencial
c) feocromocitoma
d) Preeclampsia

La hipertensión es la complicación médica más común del embarazo , aunque para


algunos autores es la segunda complicación médica del embarazo sólo después de la
anemia; es más frecuente en jóvenes durante el primer embarazo y en nulíparas de
mayor edad, hipertensas previas y diabéticas.
En México, también es la complicación más frecuente del embarazo, la incidencia es
de 47.3 por cada 1 000 nacimientos y es además, la primera causa de ingreso de
pacientes embarazadas a las unidades de terapia intensiva (debido a hemorragia
masiva, para recibir soporte hemodinámico), según la secretaría de salud (2001) la
mortalidad por complicaciones del embarazo ocupa el 15º lugar en la mortalidad
hospitalaria en general. Además, la tasa de preeclampsia se ha incrementado 40% en
el periodo entre 1990 y 1999 y constituye hasta 40% de los partos prematuros
iatrogénicos.
.

Preeclampsia
La preeclampsia es un síndrome clínico caracterizado por hipertensión con disfunción
orgánica múltiple, proteinuria, edemas.
Es definida como un incremento de al menos 140/90 mmHg después de la semana 20
de gestación, un incremento en la presión sanguínea diastólica de al menos 15 mmHg
respecto a un nivel previo a la semana 20 combinado con proteinuria (> 300 mg en 24
horas). Las mediciones de la presión arterial citadas deben ser medidas al menos 2
ocasiones con por lo menos 6 horas de separación. La proteinuria puede ser una toma
simple de orina al azar que indique al menos 30 mg/dL 3 ó ++ en dos muestras de
orina1 según el tipo de prueba. El criterio del incremento de 30 mmHg en la presión
sistólica y/o 15 mmHg en la presión diastólica respecto a valores previos a la semana
20 de gestación ha sido eliminado por ser poco específico15
Myers JE, Baker PN. Hupertensive diseases and eclampsia. Curr Opin Obstet Gynecol
2002; 14: 119-125
2. Tierney, McPhee, Papadakis. Diagnóstico clínico y tratamiento 2003. 38ª ed,
México, Manual Moderno, 2003: 770-773
3. Wilson MI, Goodwin TM, Pan VI, Ingles SA. Molecular epidemiology of
preeclampsia. Obstet and Gynecol Survey 2003; 58(1):39-66
4. Burrow GM. Complicaciones médicas Durante el embarazo. 4ª ed, México, McGraw-
Hill panamericana: 1996: 1-25
5. Guyton AC, Hall JE. Embarazo y lactancia en: Tratado de fisiología médica, 10ª ed,
México, McGraw-Hill Interamericana 2001: 1135-45

44.- Femenino de 29 años. Acude al servicio de urgencias por presentar salida de


líquido vaginal. Antecedentes: G2, P1, cursa embarazo de 36 semanas de gestación
exploración física: cervix con 10% de borramiento, 1 cm de dilatación y Tarnier
positivo.
La complicación más frecuente en esta paciente es:

a) Corioamnioitis.

b) Parto pre término.

c) Sepsis neonatal.

d) Endometritis.

Corioamnioitis: El diagnóstico de la infección intraamniótica (IIA) es básicamente


clínico. La corioamnionitis se debe descartar en toda gestante que presente fiebre sin
foco aparente, sobre todo si se sospecha o se ha confirmado una rotura de
membranas. Los criterios más empleados para el diagnóstico son: fiebre materna y, al
menos, 2 de los siguientes signos: taquicardia materna, taquicardia fetal, irritabilidad
uterina, leucocitosis materna o líquido amniótico purulento o maloliente.
Progresos de obstetricia y ginecología: revista oficial de la Sociedad española de
ginecología y obstetricia, ISSN 0304-5013, Vol. 48, Nº. 6, 2005 , pags. 316-317

45.- Una mujer de 32 años de edad es llevada a quirófano por una laparoscopía
diagnóstica debido a dolor pélvico y en cuadrante superior izquierdo crónicos (durante
los últimos 2 años). No tiene alteraciones funcionales vesicales o intestinales. Tiene
antecedente de 2 episodios de gonorrea previos. Bebe una cerveza al día. Labs: HCG
urinaria negativa; Hto 39%; Leuc. T 8 000; Plt 200 000; AST 12; ALT 14.
Intraoperatoriamente se observan adhesiones densas que involucran los oviductos,
ovarios y útero. También se observan adhesiones perihepáticas que se extienden
desde la superficie hepática hacia el diafragma. De los siguientes es el diagnóstico
más probable es:

a) Sx de Fitz-Hugh-Curtis
b) Hepatitis
c) Carcinoma hepatocelular
d) Sx Wolff-Parkinson-White

El síndrome de Fitz-Hugh-Curtis se define como la presencia de una perihepatitis


asociada a salpingitis. Los agentes etiológicos reconocidos hasta la fecha son
Chlamydia trachomatis y Neisseria gonorrhoeae. El cuadro clínico de este síndrome es
inespecífico y puede ser confundido con procesos inflamatorios o infecciosos del tubo
digestivo, aparato urinario y respiratorio, en los cuales la manifestación sintomática
fundamental es el dolor en hipocondrio derecho. El diagnóstico debe de sospecharse
en aquella mujer joven con vida sexual activa que tenga antecedentes de
promiscuidad en ella o en su pareja, que se queje de dolor subcostal derecho. Es más
probable el diagnóstico si se cuenta con el antecedente de enfermedad pélvica
inflamatoria y más aún, si se tiene evidencia de que ésta sea causada por Neisseria
gonorrehoeae y/o Chlamydia trachomatis. El diagnóstico definitivo se realiza con la
visualización directa de la adherencia perihepática por laparoscopía o laparotomía. Se
recomienda la primera. El tratamiento médico es a base de cefalosporinas y
dicloxacilina y en algunos casos se requiere de la extirpación quirúrgica del proceso
adherencial para mitigar el dolor.

El Síndrome de Fitz-Hugh-Curtis. Causa frecuente de error de diagnóstico en


hepatología y gastroenterología / The Fitz-Hugh-Curtis Syndrome. a frequent
misdiagnosis in hepatology and gastroenterology
Rev. gastroenterol. Méx;60(4):223-8, oct.-dic. 1995.

46.- Femenino de 25 años, con embarazo de término, sin antecedentes de control


prenatal. G 3. C-1. Se ingresa al servicio de obstetricia por presentar actividad uterina
regular y dolorosa. Ef.: Deambulante, tranquila, adecuada coloración de tegumentos,
abdomen con fondo uterino a 32 cm. con producto único vivo en situación transversa
dorso inferior FCF 144, al tacto vaginal cérvix dilatado a 3 cm. y membranas íntegras.
Se realiza cesárea con retención de placenta e invasión a vejiga.
La alteración placentaria que presenta esta paciente es:

a) placenta increta
b) placenta acreta
c) placenta percreta
d) placenta marginal

 Es la penetración y adherencia anormal de la placenta en la pared uterina.

 Se divide en:
 Placenta acreta.
 Placenta increta.
 Placenta percreta.

 ACRETA: Las vellosidades se adhieren al miometrio.

 INCRETA: Penetran más de la mitad del espesor del miometrio.

 PERCRETA: Atraviesa todo el espesor del miometrio, llegando


a la serosa, incluso atravesándola y adhiriéndose a órganos
vecinos.

Factores:
 Endometriósis previa.
 Tumores submucosos. (Miomas)
 Cicatríz uterina previa. (Cesárea, miomectomía)
 Implantación baja. (Placenta previa)
 Malformaciones placentarias. (Placenta extracorial)
 Legrado enérgico previo.
 Extracción manual previa de una placenta.

Diagnóstico transparto-:
 Placenta retenida por más de 20 minutos.
 Imposibilidad para encontrar un plano de separación placentaria cuando se
intenta su extracción manual.
 Hemorragia incontrolable después de la pseudoextracción.
 El diagnóstico histopatológico corrobora el diagnóstico clínico.
 Escenario menos deseable.

Tratamiento:
 Histerectomía Obstétrica.
 Constituye una cirugía no planeada y secundaria al hallazgo del
acretismo placentario con sangrado incohercible.

 Cesárea-Histerectomía. (Con diagnóstico previo)


 Cirugía planificada ante un correcto diagnóstico prenatal.

 Recomendación ACOG:
 Maduración pulmonar intrauterina.
 Inyectar al cordón umbilical 50 mg de metrotexate.
 Ligar el cordón en el nacimiento placentario y dejar la placenta in-situ.
 Embolización inmediata de arterias uterinas bilaterales, así como de
ramas de la división anterior de la arteria iliaca interna con alcohol
polivinílico.
 Continuar con 5 dosis I.M. de 50 mg de metrotexate y cuantificar niveles
de βhCG.
 Programar Histerectomía Total Radical Abdominal y/o Cistectomía
parcial y/o resección pared anterior recto.

Lee et al. Conservative Management of Placenta Percreta. Obstet Gynecol,


112(2):421-424

47.- Femenino de 21 años de edad refiere irregularidades menstruales tipo hipo-opso-


oligomenorrea desde hace 3 años. Niega tener vida sexual activa y no recuerda su
fecha de última menstruación. No hay antecedente de galactorrea ni de uso de
hormonales exógenos. Talla 164cm, peso 60kg. Sin datos de hirsutismo, las mamas,
útero y anexos son normales. El ultrasonido pélvico es normal, así como el perfil
hormonal. El diagnóstico clínico más probable es:

a) Hiperplasia del endometrio

b) Sangrado uterino disfuncional

c) Alteración menstrual fisiológica


d) Endometriosis

El ciclo menstrual normal ha cambiado en el curso de los siglos, dependiendo de las


modificaciones en el patrón reproductivo y el estado nutricional de la población.

El debut del sangrado menstrual, definido como menarquia, marca una etapa
importante en la madurez biológica durante la pubertad.

En el Programa Nacional de Atención Integral a la Salud de los Adolescentes se


mencionan los trastornos menstruales como una de las primeras causas de consulta o
de urgencia en servicios clínicos de primero y segundo nivel.

Diversas alteraciones menstruales, ya sea las que inducen aumento en la cantidad o


frecuencia o las que se presentan como episodios infrecuentes e irregulares se
observan en la adolescencia y pueden impactar la calidad de vida de las pacientes,
con repercusiones significativas en el ámbito reproductivo y metabólico.

Por esas razones podemos afirmar que los trastornos menstruales constituyen un
problema de salud pública y requieren un enfoque más integral en aspectos
preventivos y curativos en la etapa de la adolescencia.

Aproximadamente la mitad de todas las adolescentes tienen períodos irregulares


durante el primer año después de la menarquia. Estos periodos irregulares pueden
persistir hasta cinco años después de la menarquia en 20 % de estas adolescentes. La
principal complicación es la anemia, que puede ser severa y raramente tiene
consecuencias fatales.

PATRÓN MENSTRUAL NORMAL

La Federación Internacional de Ginecología y Obstetricia considera el patrón


menstrual normal entre tres y cuatro días, aunque fluctúa entre dos y siete días. El
intervalo entre menstruaciones es de veintiocho días, considerándose como límites de
veintiuno a treinta y cinco días.

El volumen de sangre menstrual es de sesenta a ochenta mililitros; no obstante, se


consideran normales los rangos de sangrado entre cincuenta y ciento cincuenta
mililitros. El aspecto de la sangre es rojo oscuro incoagulable.

En la práctica médica se utiliza un grupo de términos para hacer referencia a las


diversas alteraciones del ciclo menstrual, que requieren precisión por la frecuencia en
que son diagnosticados.

Según Schiavon (2000), las alteraciones menstruales más frecuentes son:


• oligoamenorrea: episodios de sangrado infrecuentes, irregulares, con intervalo de
más de cuarenta días;
• polimenorrea: episodios frecuentes pero regulares de sangrado uterino, que ocurren
a intervalos menores de veintiún días;
• menorragia: sangrado excesivo, tanto en cantidad como en duración, que ocurre con
regularidad y es sinónimo de hipermenorrea;
• metrorragia: sangrado generalmente no excesivo, que ocurre a intervalos irregulares;
• menometrorragia: sangramiento generalmente excesivo y prolongado, que ocurre a
intervalos frecuentes e irregulares;
• hipomenorrea: sangrado uterino regular, pero disminuido en cantidad;
• sangrado intermenstrual: sangrado uterino generalmente no excesivo, que ocurre
entre períodos menstruales regulares.

Las irregularidades menstruales son causa frecuente de consulta en las adolescentes,


siendo 95 % de las veces de naturaleza disfuncional, por inmadurez del eje
hipotálamo–hipofiso–ovárico (HHO).

Uno de los primeros problemas que hay que plantear ante estas irregularidades
menstruales, es la hemorragia uterina disfuncional (HUD). Su definición guarda
relación con las características en cantidad y frecuencia que difieren del sangrado
menstrual normal.

Con mayor frecuencia se encuentra en forma de sangrados excesivos y prolongados,


asociados a ciclos anaovulatorios, en ausencia de una patología o enfermedad
existente, aunque raramente la HUD puede presentarse con ciclos ovulatorios.

De forma práctica, consideramos una hemorragia uterina (HU) como anormal cuando
el sangrado es excesivo, con cualquier desvío o alteración de su duración, cantidad o
intervalo.

El diagnóstico de HUD supone una alteración de origen endocrino (eje HHO); por lo
tanto, su diagnóstico impone haber descartado cualquier patología orgánica y
sistémica que produzca hemorragia genital. Es un diagnóstico por exclusión.

ETIOPATOGENIA DE LA HUD EN LA ADOLESCENCIA

Los ciclos anovulatorios son más frecuentes en las adolescentes por la inmadurez del
eje HHO en el primer año tras la menarquia.

En esos casos de HUD, como ya se mencionó, se producen ciclos anovulatorios que


se traducen en una proliferación desorganizada del endometrio por falta de efecto
progestagénico. Una vez que el endometrio alcanza un grosor crítico, comienza a
descamarse en forma irregular, traduciéndose en un sangrado permanente de cuantía
variable.

Las manifestaciones clínicas de la HUD son:


• fases de amenorrea de dos a cuatro meses, seguidas de salida de sangre abundante
durante tres o cuatro semanas; en oportunidades existe irregularidad completa en el
sangrado;
• sangrado de más de seis compresas (bien empapadas) al día;
• presencia de coágulos;
• suele ser indolora;
• menstruaciones de más de siete días de duración;
• ciclos de menos de veintiún días.

La gravedad de esta hemorragia se clasifica, de acuerdo con el grado de anemia que


produzca, en metrorragia leve, moderada o grave:
Leve:
• metrorragia leve y prolongada,
• ciclo menstrual acortado,
• hemoglobina y hematocrito normales.
Moderada:
• metrorragia copiosa prolongada,
• ciclo menstrual acortado,
• anemia leve (cifras de hemoglobina inferior a diez gramos por litro).
Grave:
• metrorragia copiosa prolongada,
• ciclo acortado e irregular,
• anemia grave (cifras de hemoglobina de ocho gramos por litro o menos).

Ante un sangramiento uterino en estas edades se debe realizar el diagnóstico


diferencial con:
a) gestación y problemas relacionados con ésta, tales como abortos y gravidez
ectópica;
b) coagulopatía: 20 % de las adolescentes con hemorragia uterina tienen un defecto
de la coagulación. La manifestación más precoz de alteraciones de la coagulación
sanguínea puede ser evidenciada por un sangramiento genital anormal, lo cual puede
estar relacionado con deficiencias de plaquetas, leucemias, púrpuras, enfermedad de
Von Willebrand, deficiencia de protrombina u otros factores de la coagulación;
c) malformaciones del aparato genital, traumatismosgenitales, presencia de cuerpos
extraños;
d) dispositivos intrauterinos;
e) tumores uterinos, sarcoma botroides o tumores anexiales;
f) hipo o hipertiroidismo;
g) insuficiencia renal o hepática.

Como el diagnóstico de HUD es de exclusión, hay que hacer una historia clínica
minuciosa, exámenes complementarios y sin falta descartar las otras causas de
sangramiento transvaginal.

Se debe precisar con detalle el nivel de desarrollo puberal, la actividad sexual y la


presencia de situaciones concomitantes como: a) contacto sexual sin protección
contraceptiva;
b) uso irregular de anticonceptivos orales o antecedentes de inserción de dispositivos
intrauterinos;
c) ejercicios físicos extenuantes;
d) historia previa de sangrado excesivo, asociado a extracciones dentarias, pequeñas
heridas, epistaxis y otras;
e) dolencias renales u hepáticas preexistentes.

Ante cualquier demanda de atención por adolescentes con sangramiento genital con
las características descritas, se requiere de un examen físico general que incluya
exploración general completa, toma de tensión arterial y pulso, búsqueda de
exoftalmia, fascie Cushing, visceromegalias o presencia de masas abdominales
palpables, edemas parpebrales y de miembros superiores, así como puntos
hemorrágicos en epidermis y otros signos de coagulopatía.

El examen ginecológico debe realizarse en todas las adolescentes, con excepción de


las que no han tenido actividad sexual y presentan sangramiento leve. Además del
examen de sus genitales, hay que efectuar una valoración citológica y microbiológica
en particular en quienes presenten manifestaciones clínicas.

En la inspección de los genitales durante el examen de la paciente, es importante


evaluar que el sangramiento se origine en lesiones ubicadas en los genitales externos,
uretra u hemorroides, así como indagar acerca de la posibilidad de abuso sexual. Si
después del tratamiento de la HUD leve continúa el sangrado, se recomienda la
realización de ecografía abdominal para precisar el diagnóstico.

En pacientes que ya han tenido relaciones sexuales se debe buscar si el útero tiene
características gravídicas, la posibilidad de un aborto en curso, así como la presencia
de una masa anexial que permita corroborar la existencia de embarazo o alguna
neoplasia benigna o maligna. El examen con espéculo podrá demostrar un cuello
hiperémico, sangrante o gravídico; de encontrarse estos hallazgos, se descartaría la
etiología disfuncional del sangrado.

Exámenes complementarios básicos que no pueden faltar:


• hemograma completo,
• coagulograma completo,
• orina,
• ultrasonido ginecológico abdominal, transvaginal o transrectal según proceda,
• ecografía abdominal. Si fuese necesario por los signos identificados en el examen de
la paciente, se deben realizar:
• dosificación de FSH, LH, T3, T4, TSH y prolactina si hay sospechas clínicas de otras
enfermedades endocrinas concomitantes,
• laparoscopia en casos seleccionados por patología de base,
• otros, según hallazgos de la historia clínica y la exploración.

48.- Mujer que acude por amenorrea de 7 semanas asintomática, con antecedentes de
enfermedad pélvica inflamatoria tratada anteriormente, sin evidencia mediante
ecografía de útero ocupado. Al determinar la B-HCG, se obtiene cifra de 2,500UI/l.
Ante estos datos, el diagnóstico más probable es:

a) Gestación de evolución normal correspondiente a amenorrea.


b) Gestación ectópica asintomática.
c) Aborto precoz completo con expulsión total de restos ovulares intrauterinos.
d) Embarazo molar de inicio, sin signos ecográficos intrauterinos.

FACTORES DE RIESGO
 Anomalías tubarias

 Enfermedad pélvica inflamatoria 30-50%


 Adherencias (endometriosis)
 Antecedente de salpingoclasia
 Alteraciones anatómicas
EMBARAZO ECTÓPICO

< 6500mUI

USG normal

Cuantificación seriada
de HGC

Estabilización o
Descenso Aumento
incremento

Dilatación y legrado Laparoscopia Dx Repetir USG


64 02/07/2013

Diagnóstico:

 HGC > 1500 U/L


 Sin evidencia de saco
 Líquido libre en fondo de saco
 Sensibilidad 63%
 Especificidad 100%

Ankum WM., Mol Bw.,Van der Veen F.,Bossuyt PM.


Risk factors for ectopic pregnancy:a meta-analysis. Fertil Steril :65:1093-9.

49.- Femenino de 39 años con embarazo de 32 semanas. Ingresa al servicio de


urgencias por presentar cuadro de 1 día de evolución por sangrado transvaginal leve,
rojo brillante, sin contractilidad uterina. A.G.O.: G-3, P-0, C-2. E.F.: IMC: 30 kg/m2,
F.U. de 28 cm, fcf presente, especuloscopía cérvix cerrado con huellas de sangrado.
USG reporta producto único vivo con fetometría normal, líquido amniótico normal y
placenta anterior que cubre parcialmente el orificio cervical interno.

El siguiente paso en la atención de esta paciente es:

a) manejo por consulta externa y vigilar sangrado


b) manejo por consulta externa con uteroinhibidores
c) ingreso a hospital e inductores de maduración pulmonar
d) ingreso a hospital y cesárea.
50.- Se trata de femenino de 31 años nuligesta pero con actividad sexual regular, sin
método de planificación familiar, con ciclos regulares, sin leucorrea, refiere
dispareunia profunda, sangrado intermestrual y dismenorrea secundaria
ocacionalmente presenta urgencia urinaria, el método de elección para confirmar el
diagnóstico clínico de ésta paciente es:

a) Cultivo de exudado vaginal


b) Ultrasonido
c) Biopsia de endometrio
d) Laparoscopia con biopsias dirigidas
Endometriosis

Examinación bimanual
Dolor en los ligamentos utero‐sacros
Nodularidad en el fondo de saco
Induración del septo rectovaginal
Útero en retroversión fijo
Masa anexial

Endometriosis

Laparoscopia
visualización directa
estadificación
toma de muestras

Estadio I Minimo
Estadio II Leve
Estadio III Moderado
Estadio IV Severo
Referencias bibliográficas

1. Ruiz V. Endometriosis y fertilidad. Ed. Acosta y Warman, pp. 99

4. Acosta AA. Buttram VC Jr. Besch PK, Malinak LR, Van Der Heyden J. A.proposed
classfication of pelvic endometriosis. Obstet Gynecol

5. Buttran VC Jr. Evolution of the revised American Fertility classification of


endometriosis. Fert. Steril

51.- Paciente que acude a revisión por 8 semanas de amenorrea, asintomática. En la


ecografía vaginal se observa una tumoración anexial izquierda, sin evidencia de
embrión. En el útero no se visualiza saco gestacional. La actitud más correcta ante
este caso es:

a) Ingreso para legrado.


b) Culdocentésis.
c) Salpingectomía urgente.
d) Ingreso y determinación de niveles de B-HCG.

FACTORES DE RIESGO
 Anomalías tubarias

 Enfermedad pélvica inflamatoria 30-50%


 Adherencias (endometriosis)
 Antecedente de salpingoclasia
 Alteraciones anatómicas

EMBARAZO ECTÓPICO

< 6500mUI

USG normal

Cuantificación seriada
de HGC

Estabilización o
Descenso Aumento
incremento

Dilatación y legrado Laparoscopia Dx Repetir USG


64 27/01/2014

Diagnóstico:

 HGC > 1500 U/L


 Sin evidencia de saco
 Líquido libre en fondo de saco
 Sensibilidad 63%
 Especificidad 100%

Ankum WM., Mol Bw.,Van der Veen F.,Bossuyt PM.


Risk factors for ectopic pregnancy:a meta-analysis. Fertil Steril 2005:65:1093-9.

52.- Se trata de femenino de 26 años acude a urgencias, G4 A3, cursa con embarazo
de 17 semanas, refiere sangrado transvaginal con salida de líquido transvaginal. Se
realiza ultrasonido que reporta feto de 17 semanas con latido cardiaco y movimientos
presentes, líquido amniótico considerablemente disminuido. A la EF TA 110/80 mm hg
FC 98x´FR 22x´T 37.8 °C. Salida de líquido transvaginal, cérvix corto blando posterior.

La conducta terapéutica más apropiada en este caso es:

a) cerclaje de urgencia + antibiótico de amplio espectro


b) cristalografía
c) solicitar exámenes de laboratorio y manejo expectante
d) administración de misoprostol + antibiótico de amplio espectro

La detección de membranas rotas (RPM) antes de las 22 semanas con feto vivo se
cataloga como aborto inevitable, procediéndose a la evacuación uterina y uso de
antibióticos para evitar la sepsis. El aborto incompleto, inevitable, diferido y en
evolución puede ser manejado únicamente con prostaglandinas (misoprostol),
teniendo la misma eficacia via oral o vaginal. Se deberá mantener a la paciente
hospitalizada y vigilar la presencia de fiebre, sangrado profuso.
Current Obstetric & Gynecologic Diagnosis & Treatment Ninth Ed.
DeCherney, Alan H., et al, Ch. 14. MedlinePlus [Internet]. Bethesda (MD):
National Library of Medicine (US); [updated 2006 Feb 23]. Pregnancy Loss;
[updated 2006 Feb 22; reviewed 2006 Feb 7; cited 2006 Feb 23]. Available
from: http://www.nlm.nih.gov/medlineplus/pregnancyloss.html Planning Your
Pregnancy and Birth Third Ed. The American College of Obstetricians and
Gynecologists, Ch. 15. Williams Obstetrics Twenty–Second Ed. Cunningham,
F. Gary, et al, Ch. 9.

53.- Se trata de mujer de 34 años, es atendida en sala de partos secundario a eutocia,


durante la reparación de la episiotomía media hay un marcado incremento en el
sangrado transvaginal. La causa más probable de este fenómeno es:

a) Retención de restos placentarios

b) Laceración vaginal

c) Laceración cervical

d) Atonía uterina
La mortalidad materna es un indicador de disparidad social y económica. Cada año en
todo el mundo mueren cerca de 600,000 mujeres, entre 15 y 49 años. Como resultado
de complicaciones relacionadas con el embarazo, el parto y el puerperio. Entre las
causas principales destacan: hemorragia postparto (25 %), se psis (15%), eclampsia
(12 %) y labor prolongada o detenida (8 %).1
1

DEFINICIÓN
Se define la hemorragia posparto (HPP) como la pérdida sanguínea de 500 mL. o más
en las primeras 24 horas después del parto o el descenso del hematocrito en un 10 %
o más.
CLASIFICACIÓN
HPP Inmediata.- Pérdida sanguínea de 500 mL. o más originada en el canal del parto
dentro de las 24 horas posteriores al parto.
HPP Tardía.- Sangrado anormal o excesivo originado en el canal del parto que se
presenta entre las 24 horas posteriores al parto y el final del puerperio (42 días).
FACTORES DE RIESGO
Se han descrito los siguientes factores de riesgo para la HPP:
• Embarazo múltiple
• Polihidramnios
• Macrosomía
• Trabajo de parto disfuncional
• Gran multiparidad
• Corioamnionitis
• Uso inadecuado de oxitócicos
• Endometritis en el puerperio
• Púrpura trombocitopénica
• Anestesia general
• Administración previa y reciente de inhibidores uterinos
• Placenta previa
• Enfermedad de von Willebrand
• Desprendimiento prematuro de placenta
• Acretismo placentario
CAUSAS DE HEMORRAGIA POSPARTO
A continuación se lisian las causas más frecuentes de HPP. Según su origen, se
dividen en uterinas y no uterinas:
2
Uterinas - No Uterinas
• Hipotonía o atonía uterina
• Retención de placenta o restosplacentarios o membranas
• Placentación anormal (acretismo)
• Inversión uterina
• Traumatismo uterino (rotura uterina, desgarro cervical)
• Laceraciones del canal del parto, incluyendo la episiotomía
• Coagulopatías
• Hematomas
CUADRO CLÍNICO
La HPP se caracteriza por los siguientes signos y síntomas:
• Sangrado transvaginal de moderado a grave.
• Útero flácido (no contraído) o desgarros.
• Alteraciones hemodinámicas que se manifiestan como: mareos, sudoración,
náuseas, taquicardia y/o hipotensión arterial.
• Oliguria.
• Útero nacido (no contraído).
Al considerar la HPP, deben diferenciarse dos tipos de situaciones: la prevención
orientada a minimizar la probabilidad de que una mujer presente hemorragia tras el
parto y el manejo o tratamiento de la hemorragia, cuando ésta ya se haya producido

Referencias Bibliográficas

1. AbdaRabbo SA: Stepwise uterine devascularization: A novel technique for


management of uncontrollable postpartum hemorrhage with preservation of the

4. B-Lynch C, Coker A, Lawal A II, Abu J, Cowen MC: The B-Lynch surgical
technique for the control of massive postpartum haemorrhage: an alternative
to? Five cases reported. Br J Obstet Gynaecol 17;104:372-5.

5. Brahaems D: Unwanted hysterectomies. Lancet ;342-61.

54.- Paciente femenino de 24 años, G-1, que cursa con embarazo de 37 semanas de
gestación, presenta pérdida del estado de alerta posterior a crisis convulsivas tónico-
clónicas, signos vitales con T-A 170.120mmhg Fc 95x´, reflejos osteotendinosos
aumentados, se aprecia una Fc fetal de 132x´ y edema importante de miembros
inferiores, no se aprecian datos de trabajo de parto ni modificaciones cervicales, El
diagnóstico más probable es:

a) Crisis epileptica de gran mal


b) Pre eclampsia severa
c) Hipertensión inducida por el embarazo
d) Eclampsia

CUADRO 1. DIAGNÓSTICO*
Preeclampsia Leve: Se presenta después de la semana 20 de gestación, durante el
parto, o en las primeras 6 semanas después de éste
Presión sistólica ≥ a 140 mm Hg o presión diastólica ≥ 90 mm Hg
Proteinuria ≥ a 300 mg / orina de 24 hrs o su equivalente en tira reactiva
Preeclampsia Severa: Se presenta después de la semana 20 de gestación, durante
el parto, o en las primeras 6 semanas después de éste
Presión sistólica ≥ a 160 mm Hg o presión diastólica ≥ 110 mm Hg
Proteinuria ≥ a 2 gr en orina de 24 horas o su equivalente en tira reactiva
Creatinina sérica > a 1.2 mg/dl
Trombocitopenia ≤ 150 000 cel/mm3
Incremento de la deshidrogenasa láctica ≥ a 600 UI
Elevación al doble de los valores de TGO/AST o TGP/ALT
Cefalea, alteraciones visuales o auditivas
Epigastralgia
Oliguria ≤ a 500 ml en 24 horas 7
Edema agudo de pulmón
Dolor en hipocondrio derecho
Restricción en el crecimiento intrauterino
Oligohidramnios
Eclampsia Preeclampsia mas convulsiones sin otra causa. Se presenta después de la
semana 20 de gestación, durante el parto, o en las primeras 6 semanas después de
éste.
Síndrome de HELLP Criterios para establecer el diagnóstico del síndrome de HELLP:
Plaquetas < 100 000/mm3 TGO/AST ≥ 70U/L DHL ≥ 600U/LBilirrubina total > 1.2
mg/dl
Se presenta después de la semana 20 de gestación, durante el parto, o en las
primeras 6 semanas después de éste.
Hipertensión Crónica: Se diagnostica cuando existe hipertensión arterial ≥ a 140/90
mm Hg antes de la semana 20 de gestación o si persiste después de doce semanas
posteriores al parto.
Las pacientes con hipertensión crónica deben ser evaluadas antes del embarazo para
determinar la severidad de la hipertensión y facilitar la planeación de un embarazo
mediante el cambio de medicamentos y de hábitos higiénicos y dietéticos para evitar
complicaciones.
Hipertensión Gestacional: Presencia de hipertensión arterial ≥ a 140/90 mm Hg
después de la semana 20 de gestación y se mantiene hasta las doce semanas
después del parto
Ausencia de proteinuria
Presencia o no de cefalea, acúfenos y fosfenos
Después de 12 semanas de la interrupción del embarazo se revalorará la presencia de
hipertensión, si continúa, se reclasifica como hipertensión crónica: es un diagnóstico
retrospectivo. 8 Si no hay, se clasifica como hipertensión transitoria.

1. Aagaard-Tillery KM, Belfort MA. Eclampsia: morbidity, mortality, and management.


Clin Obstet Gynecol 48:12-23, 2005.
2. Atallah AN, Hofmeyr GJ, Duley L. Calcium supplementation during pregnancy for
preventing hypertensive disorders and related problems. Cochrane Database Syst Rev
1:CD001059, 2001.
3. Barton JR, Sibai BM. Diagnosis and management of hemolysis, elevated liver
enzymes, and low platelets syndrome. Clin Perinatol 31:807-33, 2004.
4. Baxter JK, Weinstein L. HELLP syndrome: the state of the art. Obstet Gynecol Surv
59:838-45, 2004.
5. Cetin A. Eclampsia. In Mohler III ER, Townsend RR. Advanced therapy in
hypertension and vascular disease. Ontario: B.C. Decker Inc. pp. 407-15, 2006.
6. Cetin A. Hemolysis, elevated liver enzymes, and low platelets (HELLP). In Mohler III
ER, Townsend RR. Advanced therapy in hypertension and vascular disease. Ontario:
B.C. Decker Inc. pp. 416-20, 2006.
7. Chappell LC, Seed PT, Briley AL, Kelly FJ, Lee R, Hunt BJ, Parmar K, Bewley SJ,
Shennan AH, Steer PJ, Poston L. Effect of antioxidants on the occurrence of pre-
eclampsia in women at increased risk: a randomised trial. Lancet 354:810-16, 1999.

69Silverberg SG, Kurman RJ. Tumors of the uterine corpus and gestational
trophoblastic disease. En: Rosai J, Sobin LJ, editores. Atlas of tumor pathology: tumors
of the uterine corpus and gestational trophoblastic disease, fasc. 3, ser. 3. Washington
DC: Armed Forces Institute of Pathology; 1992, p. 219-85.
55.- Femenino de 22 años de edad, acude al servicio de urgencias por referir malestar
general, fosfenos, náusea y vómito. Antecedentes: cursa con embarazo de 36.5 SDG,
G2, A1, C0. Exploración física: TA 185/110 mmHg, FC 120 lpm, FR 35 x’, T37.9°,
alerta, inquieta, aprecia ictericia, cardiopulmonar sin compromiso, abdomen globoso a
expensas de útero grávido, se detecta PUVI, longitudinal, cefálico, dorso a la derecha,
FCF 142 x’, al tacto genital cérvix, central, formado, cerrado, extracción de guantes sin
evidencia de pérdidas genitales, extremidades inferiores edema +++.
El dato de laboratorio que apoya el diagnóstico en esta paciente es:

a) Hematuria.
b) Proteinuria.
c) Coluria.
d) Piuria.

Referencia:
La preeclampsia se define por aumento de la presión arterial y la presencia de
proteinuria durante el embarazo.
1) Leve: las pacientes suelen tener unas cuantas manifestaciones, y su presión
arterial diastólica es menor de 110 mmHg. En ocasiones hay edema. La cifra
de plaquetas es mayor de 100 000/µl.
2) Grave: los síntomas son más notorios y persistentes. La presión arterial casi
siempre es con niveles mayores de 160/110 mmHg. Puede haber
trombocitopenia que avance hasta coagulación intravascular diseminada.
Se requiere hospitalización para las mujeres con preeclampsia; debe obtenerse una
biometría hemática completa con cifra de plaquetas y determinación de electrolitos,
que además incluya enzimas hepáticas. Se obtiene una muestra de orina de 24 horas
para determinar la depuración de creatinina y proteínas totales al ingreso hospitalario.
Se debe controlar la hipertensión arterial, para evitar sufrimiento fetal, así como
empeoramiento de la paciente.

Bibliografía:

1. McPhee S, Papadakis M, et. al. Diagnóstico Clínico y Tratamiento 2010. Lange,


McGraw Hill,
49ª edición, México, 2010.
2. Sibai BM, Diagnosis, prevention, and management of eclampsia. Obstet Gynecol.
2005. Feb; 105;: 402 – 410.

56.- Se trata de femenino de 28 años, es atendida en consulta por secreción láctea


bilateral hace 4 meses y menstruaciones cada 21 días, en escasa cantidad desde
hace 6 meses. Antecedentes: hace 6 meses padece gastritis tratada con cimetidina
durante 2 meses y posteriormente ha tomado el medicamento de forma irregular.
a.g.o.: menarca 12 años, ritmo 30/7, núbil. e.f.: talla 1.63, peso 54 kg. Al efectuar
compresión en glándulas mamarias se produce salida de secreción láctea bilateral,
resto normal.
El tratamiento de primera elección para esta paciente es:

a) bromocriptina.
b) omeprazol.
c) omeprazol.
d) cabergolina.

La cabergolina tiene ventaja sobre la bromocriptina en términos de eficacia y


tolerabilidad.
En el tratamiento de pacientes con prolactinoma e hiperprolactinemia idiopática,
cabergolina en comparación con bromocriptina, es más efectiva en : a) reducción del
tamaño tumoral, b) lograr normo prolactinemia y c) resolver amenorrea, oligo
menorrea.
Cabergolina representa una intervención terapéutica efectiva y bien tolerada, en
pacientes con prolactinoma gigante invasivo, al reducir los niveles de PRL y en el
tamaño del tumor.
Se recomienda utilizar cabergolina sobre otros agonistas de la dopamina.
www.cenetec.salud.gob.mx/.../imss_652_13_txfarmacologicohiperprolactinemiager.

57.- Femenino 40 años, G-3 P-2 A-1, se detecta de anemia ferropénica, de 9.5 g/dl,
refiere ciclos menstruales de 31,32 x 8,9 días de duración, acompañados de coágulos,
los cuales aparecieron después del nacimiento de su segundo hijo hace 12 años. e.f.:
con ligera palides de tegumentos, S/V dentro de los parámetros normales, genitales
con evidencia de sangrado activo, al tacto vaginal bimanual se detecta útero de
consistencia firme voluminoso, irregular, aproximadamente de 12 cm. anexos libres.
El tratamiento más apropiado para esta paciente es:

a) histerectomía total sin conservar anexos.


b) histerectomía total conservando anexos.
c) histerectomía vaginal.
d) histerectomía radical.

Tratamiento: La cirugía es el tratamiento más común del mioma uterino:


Miomectomia: Extirpar sólo el mioma, los tumores únicos y accesibles, la resección
histeroscópica de miomas submucosos también es posible realizarlo con
electrocoagulación en pacientes con hemorragias (>90%).
Histerectomía:
La cirugía puede ser abdominal o laparoscópica, la decisión final de la técnica
dependerá de la elección del cirujano, en base al caso individual y a su experiencia, el
procedimiento puede ser:
-histerectomía subtotal.
-histerectomía total (remoción cervical). Técnicamente más sencillo en su realización.
La incidencia de carcinoma con origen en el muñón cervical es menor al 1% en
nuestros días.
Bibliografía:
1. hanafi m. predictors of leiomyoma recurrence after myomectomy. am coll obstet
ginecol 2005;
105: 877-880.
2. inclan j, mojarra j. miomectomía histeroscópica. Abordaje actual para el manejo de
los miomas submucosos. Reporte de un caso y revisión de la literatura. 2001;18: 29-
34.
3. goldrath mh, husain m. the hysteroscopic management of endometrial
leiomyomatosis. j
assoc gynecol laparosc; 4: 263-267.
4. clement pb, scully re. mullerin adenofibroma of the uterus with invasion of
myometrium and pelvic veins. int j gynecol pathol 1990; 9: 363-371.
58.- Se trata de femenino de 17 años de edad que acude al servicio de urgencias,
primigesta, cursa con 31 semanas de gestación , refiere salida de líquido transvaginal,
escaso, claro. Tacto vaginal: Tarnier negativo, cérvix posterior cerrado, formado, no
hay datos de sangrado transvaginal. TA 120/85 mmHG, FC 120 lpm, FR 23 rpm, T
38.1°C.

El siguiente paso más adecuado en este caso es:

a) Realizar cristalografía
b) Realizar ultrasonografía
c) Registro cardiotocográfico
d) Biometría hemática

Las dos principales pruebas diagnósticas de ruptura prematura de membranas son la


cristalografía y la prueba con papel de nitrazina. La combinación de la historia clínica,
la prueba de nitrazina y la cristalografía debe ser usada para evaluar a las pacientes
con RPM. Esta combinación alcanza una exactitud diagnóstica del 93.1% La
realización de la ultrasonografía no ha demostrado ser una herramienta en el
diagnóstico de RPM, solo es útil para la cuantificación del líquido amniótico y la RMP
no se asocia necesariamente a oligohidramnios.
Prevención, Diagnóstico y Tratamiento de la Ruptura Prematura de Membranas
(RPM). Guía de Referencia Rápida: Guía de Práctica Clínica. México,
CENETEC; Disponible en:
http://www.cenetec.salud.gob.mx/contenidos/gpc/catalogoMaestroGPC.html

59.- Se trata de paciente femenino de 40 años portadora de DM tipo 2 tratada


actualmente con sufonilureas, acude a consulta quejándose de descarga vaginal
prurítica y blanquecina desde hace 15 días. Por los datos recabados el examen de
diagnóstico más útil para identificar el patógeno es:

a) Wet prep.
b) Tincion de Gram
c) PH
d) KOH

Prueba del KOH es un procedimiento en el cual hidróxido del potasio (KOH) se utiliza
detectar hongos disolviendo las células humanas en una cultura. La diferencia en la
composición de la pared de célula de células humanas y de células fungicidas permite
que este procedimiento ayude a distinguir las dos células. El KOH desnaturaliza las
proteínas en la célula humana; solamente sigue habiendo las células fungicidas ser
considerado debajo del microscopio.
7. Faro S. Leucorrea. Causas infecciosas e imbalances en el ecosistema vaginal.
Tribuna Médica. Cahners Healthcare ed 1996; 1 – 10.
60.- A 23-year-old nulligravid female has not menstruated in the past 4 months.
Previously, her menstrual cycles were regular. She is otherwise well and denies recent
onset of stress, change in exercise routine, headaches, visual field alterations, or
galactorrhea.. She has a body mass index of 24, blood pressure of 120/78 mm Hg, and
does not appear hirsute. No adnexal masses can be palpated. Laboratory
investigations reveal a negative beta human chorionic gonadotropin (β-hCG), normal
thyroid-stimulating hormone (TSH), and prolactin levels.
What is the next best step in the management of this patient?

a) Measurement of luteinizing hormone.

b) CT scan of the sella turca.

c) Prescribe oral estrogen for 21 days followed by 7 days of medroxyprgesterona


and reevaluate.

d) Preescribe 7 days of medroxyprgesterona and reevaluate.

La Prueba de respuesta a progestágenos se basa en la observación de que el


tratamiento con progestágeno (acetato de medroxiprogesterona 10 mg por 5 a 6 días)
solo induce la menstruación en las mujeres con concentraciones normales de
estrógenos circulantes. Una prueba positiva (hemorragia después de concluir el
tratamiento con progestágenos) señala cifras normales de producción de estrógenos y
una prueba negativa (sin hemorragia por privación), hipogonadismo franco.
Danforth, Tratado de Obstetricia y Ginecologia, 9ª Edición, Ed. Mc Graw Hill
Interamericana, Pág 668

61.- A woman presents with painless vaginal bleeding at 37 weeks´ gestation. The fetal
heart rate is stable in the 150 pbm. Which of the following is indicated?

a) Ultrasound examination
b) Nonstress test
c) Induction of labor
d) Digital examination of the cervix

Diagnóstico por USG.


Sangrados 2da Mitad Embarazo
Diagnóstico diferencial (1)
DPPNI PLACENTA PREVIA

DOLOR PRESENTE AUSENTE


HIPOGASTRICO

AUMENTADO RELAJADO
TONO UTERINO

COMPROMISO
FRECUENTE INFRECUENTE
FETAL

U.S.G. DIAGNOSTICA DIAGNOSTICA

62.- Femenino de 19 años acude a consulta por padecer un cuadro de dolor abdominal
de inicio súbito, refiere alteraciones en su ciclo menstrual. La exploración física revela
una tumoración dolorosa en el anexo izquierdo. La prueba de embarazo es negativa.
La radiografía muestra una masa opaca en la fosa ilíaca izquierda con áreas de
calcificación. El diagnóstico clínico más probable es:

a) Cistadenoma mucinoso
b) Teratoma quístico
c) Quiste folicular
d) Cistadenoma seroso

Los tumores de células germinales constituyen casi el 20% de los tumores de ovario y
de ellos un 95% son benignos, siendo el tipo más frecuente el teratoma maduro
benigno o también denominado quiste dermoide (1).

Aproximadamente el 80% se presentan en mujeres en edad fértil. Se originan a partir


de células embrionarias pluripotenciales presentes habitualmente en ovario, testículo,
mediastino, retroperitoneo y región sacrocoxígea, esto explica que la coloración de las
faneras del quiste coincida con el fenotipo del paciente (2).

El teratoma quístico benigno con relativa frecuencia es un tumor bilateral (del 7 al 25%
según los autores) (1) y se caracteriza por una cápsula gruesa, bien formada,
revestida por epitelio plano estratificado. Bajo este se pueden encontrar una variedad
de apéndices cutáneos que incluyen glándulas sudoríparas, apocrinas y sebáceas. La
cavidad se llena de los detritus de éste y sus anexos, que es de color amarillo pálido,
grasoso, espeso y suele contener pelo. Otros tejidos que se pueden encontrar son
dientes, cartílago, plexos coroideos, falanges, tejido nervioso y en ocasiones tejido
tiroideo (struma ovarii) con potencial tirotóxico o de degeneración maligna tiroidea.

La mayor parte de los quistes dermoides son asintomáticos y la forma más frecuente
de presentación son el dolor abdominal (48%) y hemorragia uterina anormal o
concomitante (15%) o aumento del volumen abdominal (15%). La rotura de un quiste
dermoide es rara, entre el 1-1,2% y constituye una urgencia quirúrgica (3).

El tratamiento es quirúrgico, siendo posible la mayor parte de las veces una resección
del mismo, respetando el resto del ovario.

La ecografía constituye el modelo diagnóstico de elección y la combinación de


ecografía con radiografía simple de abdomen proporciona un diagnóstico más exacto
en la mayoría de los casos, siendo la resonancia magnética o la TAC el que aporte el
diagnóstico diferencial.

Las Rx de quiste dermoide se caracteriza por una cápsula bastante radioopaca y el


líquido oleoso que contiene es radiolúcido, esta conjugación presenta muchas veces
una característica de aspécto radiológico de calcificación en la pared del mismo.

Cólico nefrítico, teratoma ovárico y radiografía simple de aparato urinario Romero


Pérez P, Martínez Hernández MªC.

Servicio de Urología. Policlínico San Carlos Denia (Alicante).Actas Urol Esp.


2007:31(8):936-937ACTAS UROLÓGICAS ESPAÑOLAS SEPTIEMBRE 2007

63.- Se trata de femenino de 26 años acude a urgencias, G4 A3, cursa con embarazo
de 17 semanas, refiere sangrado transvaginal con salida de líquido transvaginal. Se
realiza ultrasonido que reporta feto de 17 semanas con latido cardiaco y movimientos
presentes, líquido amniótico considerablemente disminuido. A la EF TA 110/80 mm hg
FC 98x´FR 22x´T 37.8 °C. Salida de líquido transvaginal, cérvix corto blando posterior.
El diagnóstico más probable en este caso es:

a) amenaza de aborto
b) aborto en evolución
c) aborto inevitable
d) pérdida gestacional recurrente

El aborto inevitable es la pérdida espontánea de un feto antes de la semana 20 del


embarazo (la pérdida del embarazo después de 20 semanas se llama muerte fetal),
donde no se pueden detener los síntomas y se presenta el aborto de manera
espontánea. Existe sangrado transvaginal, contracciones uterinas y dilatación del
cuello del útero. .. En este caso la paciente ya presenta ruptura de membranas lo que
lo hace inevitable o inminente
COG Practice Bulletin Number 150: Early pregnancy loss (Pérdida temprana del
embarazo). Mayo de 2015. Colegio Americano de Obstetras y Ginecólogos (American
College of Obstetricians and Gynecologists). Obstetrics and Gynecology (Ginecología
y Obstetricia). 2015;125:1258.

64.- Se trata de paciente femenino de 25 años, G.1 , en sala de expulsión en postparto


inmediato se obtiene producto de 4100 gr, APGAR 6/9, sangrado postparto
cuantificado en 1200 ml. A la EF paciente pálida diaforética con TA 90/60, FC 128x,
FR 24, T 37.2°C. La medida terapéutica más apropiada para la paciente es:
La primer línea de tratamiento médico que se debe aplicar a la paciente es:

a) Oxitocina 20 unidades o ergometrina 0.2 a 0.5mg


b) Misoprostol 1500 mcg
c) Carbetocina 100 mcg
d) Oxitocina 20 unidades en 500 ml de solución fisiológica

Lo agentes uterotónicos deben ser la primera línea de tratamiento para la hemorragia


postparto por atonía uterina: 1) oxitocina 20 unidades en 500 ml de solución glucosada
(la oxitocina se precipita en solución fisiológica), 2) ergometrina 0.2 a 0.5 mg IM dosis
única y revalorar nueva aplicación a los 10 minutos. La carbetocina se utiliza cuando
se tiene contraindicación para ergometrina y en pacientes con pobre respuesta a
oxitocina o ergometrina.
Diagnóstico y Tratamiento de la Hemiorragia Obstétrica en la Segunda Mitad
del Embarazo y Puerperio Inmediato. Guía de Referencia Rápida: Guía de
Práctica Clínica. México, CENETEC; Disponible en:
http://www.cenetec.salud.gob.mx/contenidos/gpc/catalogoMaestroGPC.html

65.- Se trata de femenino de 17 años de edad que acude al servicio de urgencias,


primigesta, cursa con 31 semanas de gestación, refiere salida de líquido transvaginal,
escaso, claro. Tacto vaginal: Tarnier negativo, cérvix posterior cerrado, formado, no
hay datos de sangrado transvaginal. TA 120/85 mmHG, FC 120 lpm, FR 23 rpm, T
38.1°C. Los datos que apoyarían el diagnóstico de corioamnioitis en esta paciente son:

a) Fiebre >38°C + FC de 120 lpm


b) Líquido transvaginal escaso claro + FC >100 lpm
c) FR >20 rpm + Líquido transvaginal escaso claro
d) Fiebre >38°C + Tarnier Negativo

En la corioamnionitis clínica las manifestaciones más frecuentes son: 1) fiebre >38°C,


2) hipersensibilidad uterina, 3) taquicardia materna >100 lpm, 4) taquicardia fetal >160
lpm, 6) líquido amniótico fétido o purulento, 7) descarga vaginal fétida o purulenta. La
presencia de temperatura >38°C, aunado a uno o más de los criterios previamente
mencionados apoyan fuertemente el diagnóstico de corioamnionitis.
Prevención, Diagnóstico y Tratamiento de Corioamnioitis en los tres niveles de
atención. Guía de Referencia Rápida: Guía de Práctica Clínica. México,
CENETEC; Disponible en:
http://www.cenetec.salud.gob.mx/contenidos/gpc/catalogoMaestroGPC.html

66.- Se trata de paciente femenino de 45 años de edad, presenta sinusorrragia de 3


meses, no refiere haber presentado alteraciones menstruales previas, por lo demás
asintomática. La patología más probable de la paciente es:

a) Adenomiosis
b) Malformaciones Müllerianas
c) Pólipo endometrial
d) Pólipo endocervical

Pólipo es toda formación sésil o pediculada que se fija a la cavidad uterina. Las
formaciones polipoideas dentro del aparato genital femenino se dividen en
endometriales y cervicales. Dentro de los pólipos cevicales se diferencian los
ectocervicales y los endocervicales; estos últimos son considerados como los
verdaderos por estar constituidos por epitelio cilíndricco. No se conocen con certeza
los factores asociados a su histogénesis (teoría más admitida: hiperplasia focal de
la mucosa endocervical por estímulo estrogénico) (1). Los pólipos cervicales se
consideran la forma más frecuente de tumor benigno de cérvix.
Constituyen de un 3 a un 10% de las consultas ginecológicas; presentan máxima
incidencia entre la 4° y 5° década de la vida. Algunos estudios demuestran que
hasta en el 56% de casos de mujeres postmenopáusicas se encuentran asociados
pólipos cervicales y endometriales; es mucho menor esta frecuencia antes de la
menopausia (2). La clínica de presentación más común es la hemorragia
intermenstrual, espontánea o durante el coito (3). También pueden ser
asintomáticos (hallazgo casual en estudio ecográfico) o, en ocasiones, alcanzar
grandes tamaños: llegan a aflorar por el orificio cervical externo; se denominan
"pólipos gigantes" cuando superan 2 cm (4). Para el diagnóstico habitualmente se
utiliza la ecografía. Los casos sintomáticos se consideran de indicación quirúrgica
por su riesgo de degeneración a adenocarcinoma cervical.
Los pólipos cervicales constituyen una patología frecuente en la consulta ginecológica,
sobre todo en mujeres mayores de 20 años que han tenido hijos y son poco comunes
antes de la menarquia. En la mayoría de los casos, solamente se presenta un pólipo,
aunque en ocasiones se pueden encuentrar 2 ó 3. La época más frecuente es al final
de su vida reproductiva y después de la menopausia. En las primeras, las lesiones
suelen ser más grandes, difíciles de tratar y con frecuencia recidivan.
La causa de los pólipos cervicales aún no se ha comprendido completamente, pero
con frecuencia son el resultado de una infección crónica, una respuesta local anormal
a los niveles de estrógeno o a una congestión local de los vasos
sanguíneoscervicales. Clínicamente se manifiesta con sangrado genital fuera de la
menstruación, sobre todo durante la relación sexual. Son benignos, no se malignizan
pero deben ser extirpados para controlar la sinusorragia.
67.- Paciente de 40 años con nódulo mamario indoloro, de bordes imprecisos. La
mamo¬grafía revela imagen nodular, con espículas en todos sus márgenes, y 10
microcalcificaciones finas, agrupadas en el interior. El diagnóstico más probable, entre
los que se citan, es:

a) Fibroadenoma.
b) Carcinoma
c) Quiste
d) Mamografía normal para la edad de la paciente.

Técnicas diagnósticas
Exploración
Masa palpable o engrosamiento unilateral. La posibilidad de que una masa palpable
en la mama sea maligna está en relación con mayor edad, postmenopausia y con las
siguientes características en el examen físico: consistencia firme, aspecto sólido,
bordes irregulares, escaso desplazamiento sobre la piel, la región costal o los tejidos
que le rodean, unilateral, no dolorosa y la presencia de adenopatías axilares. Sin
embargo, aún en ausencia de estos factores un 10% pueden ser malignas, algunas
veces una zona de engrosamiento que no llega a masa puede ser cáncer. La
coexistencia de masa y adenopatía axilar palpable debe considerarse cáncer mientras
no se demuestre lo contrario. El 90 % de las masas suelen ser lesiones benignas. Las
masas de superficie lisa y consistencia elástica están asociadas a fibroadenoma en
mujeres entre 20-30 años y a quistes en las mujeres de 30 a 40. La exploración a
realizar ante esta situación es una mamografía si hay antecedentes de cáncer de
mama y una ecografía sobre todo si existe dolor (ICSI, 2005).
Secreción por el pezón. Siempre se debe estudiar. Hay mayor riesgo de lesión maligna
en el caso de que la secreción contenga restos hemáticos y esté asociado a masa. La
citología del líquido expulsado sólo puede ser tenida en cuenta si es positiva. Está
indicado realizar mamografía y galactografía en el caso de que el exudado se presente
en un solo conducto. La presencia de secreción lechosa bilateral orienta a causa
endocrinológica se ha de realizar el diagnóstico diferencial de galactorrea (ICSI, 2005).

Dolor. Es uno de los motivos de consulta más frecuente. En ausencia de masa otros
síntomas de sospecha suele ser debida a tensión premenstrual, dolor condrocostal y a
otras causas (ICSI, 2005). Está asociado con mayor frecuencia a cambios
fibroquísticos en la mama premenopáusica.
Síntomas cutáneos. La Enfermedad de Paget afecta al pezón y areola de forma
unilateral, clínicamente muy similar a la dermatitis crónica crónica eccematosa se
asocia a un carcinoma mamario intraductal subyacente. (Fitzpatrick, 2001)
La retracción del pezón o de la piel de presentación reciente se debe evaluar
cuidadosamente. Los fenómenos inflamatorios del tipo de eritema, induración,
aumento de temperatura y dolor pueden ser indicativos de un tumor inflamatorio de
mal pronóstico. En ocasiones un tumor evolucionado puede dar lugar a un cáncer
ulcerado.

Imágenes
Mamografía
Tiene una sensibilidad y especificidad del 90%, siendo el método aislado de
diagnóstico más eficaz, aunque en mamas densas pierde sensibilidad. Utilizada para
el screening puede reducir la mortalidad del cáncer de mama en un 33%. Nos puede
dar el diagnostico, la presencia de multicentricidad o de lesiones sincrónicas.
La mamografía nos va a valorar distintos tipos de imágenes:
1: Signos primarios:
A. Masa dominante: Valorando tamaño, densidad, forma nitidez y estabilidad en el
seguimiento. Es la lesión más frecuentemente hallada.
Considerando la clínica y los datos mamográficos, ecográficos, etc. se establece la
probabilidad de malignidad del nódulo, pudiendo expresarla en las siguientes cuatro
categorías:
o Benigno
o Probablemente benigno
o Probablemente maligno
o Maligno
NÓDULO BENIGNO

NÓDULO PROBABLEMENTE BENIGNO

NÓDULO PROBABLEMENTE MALIGNO


NÓDULO MALIGNO

B. Lesiones estrelladas (o de alteración de la arquitectura): Representadas por


áreas de distorsión de la arquitectura mamaria, de bordes irregulares y que adoptan
una morfología radiada. Suele ser un signo temprano en el carcinoma de mama y es
de difícil interpretación. Posee menos valor predictivo positivo que el nódulo o las
microcalcificaciones, por lo que se recomienda biopsia quirúrgica en todos los casos
excepto en los que los antecedentes de traumatismo, cirugía previa o inflamación
permitan optar por el seguimiento de la lesión. En estos casos siempre es conveniente
haber realizado una citología con resultado negativo.
C. Microcalcificaciones: Son hallazgos frecuentes y el análisis de sus
características nos puede ayudar a diferenciar las benignas de las sospechosas y de
las claramente malignas.
Las benignas no requieren más pruebas diagnósticas complementarias, las
probablemente benignas precisas de un seguimiento mamográfico no inferior a los dos
años y en las sugestivas de malignidad, la biopsia es preceptiva. Hay que analizar las
siguientes características:
• Tamaño: Las superiores a 2 mms. se clasifican de macrocalcificaciones y
suelen ser benignas. Por debajo de los 2 mms. se denominan microcalcificaciones y
cuanto más pequeñas y agrupadas más sospechosas son de malignidad
• Morfología: Las calcificaciones malignas suelen ser heterogéneas en forma y
tamaño, puntiagudas, anguladas, irregulares, en "coma", ramificadas y con forma de
punto y raya. Las benignas suelen ser homogéneas, redondas y en ocasiones
anulares y de centro claro.
• Número: Se considera que cuando hay cinco o más calcificaciones menores de
1 mm. en un área de 1x1 cm. de mamografía, existe sospecha de malignidad. Cuanto
mayor es el nº de calcificaciones en esa área, más sospechosas son.
• Distribución: Las calcificaciones distribuidas de forma segmentaria, no al azar,
son sospechosas e indicativas de biopsia.
• Variación en el tiempo de las calcificaciones: Las calcificaciones malignas
varían con el tiempo. La estabilidad de las calcificaciones durante año y medio - dos
años, se consideran como benignas.
• Calcificaciones asociadas a mama: Los carcinomas de mama calcifican en un
50 %. Cuando hay calcificaciones internas en lesiones con signos de malignidad,
aumentan las posibilidades de malignidad. Se hallan en un 75% de los cánceres
ocultos y suponen el 30-47% de hallazgo aislado en los cánceres de mama. Son el
primer marcador de cáncer de mama en las mujeres jóvenes.
2: Signos secundarios:
1. Engrosamiento de la piel.
2. Permeabilidad linfática.
3. Aumento de la vascularización.
4. Afectación linfática.
5. Dilatación ductal.

Bibliografía
Apantaku LM. Breast cancer diagnosis and screening. [Internet]. American
Family Physician; 2000 [ acceso 28/6/2007]. Disponible en:
http://www.aafp.org/afp/20000801/596.html
Barratt A, Howard K, Irwig L, Salkeld G and Houssami N. Model of outcomes of
screening mammography: information to support informed choices. BMJ 2005;330;936-
940.[Texto completo]
Brewer NT, Salz T, Lillie SE. Systematic review: the long-term effects of false-
positive mammograms. Ann Intern Med. 2007;146(7):502-10 [PubMed] [Texto
completo]
Breast Imaging reporting and data system (BI-RADS). 2nd ed. Reston (VA):
American College of Radiology; 2007 [Reseña]

68.- Femenino de 55 años, se queja de presión pélvica y una masa en la entrada


vaginal. Anetecedentes: G.3 P.3, el último con peso al nacer de 4,500 grs. FUR hace
tres años. Sin terapia de reemplazo. Historia de tabaquismo positivo a razón de 40
cajetillas año. Actualmente refiere dificultad para evacuar, tos crónica, presenta una
orina de 60 cc. En la exploración pélvica, el hallazgo clínico más probable es:

a) Rectocele
b) Cistocele
c) Enterocele
d) Uretrocele

El Rectocele es una hernia de la pared anterior del Recto hacia la porción posterior de
la vagina. La incidencia real de esta “deficiencia anatómica” es desconocida y en
muchas ocasiones es un resultado del paso del tiempo. Es un hallazgo muy frecuente
del examen perineal, siendo en múltiples ocasiones asintomático.
El rectocele puede ser un hallazgo importante del síndrome de Obstrucción
Defecatoria
(SOD). No debe tomarse como una deficiencia anatómica única, sino como parte
importante de un problema anatomo-fisiológico complejo.

Un principio importante es la etiología y anatomo-patología del rectocele. Existen


varias teorías sin consenso principal. La existencia, deficiencia o alteraciones del septo
rectovaginal son controversias importantes. No existe una fascia visceral que separe el
recto de la vagina o que forme un septo específico. Existe frontera entre donde termina
la pared anterior del recto y donde comienza la pared de la vagina, pero múltiples
estudios no han encontrado un septo rectovaginal específico. El septo puede estar
formado de una película casi transparente hasta una pared de consistencia
fibromuscular fuerte.
El rectocele es un hallazgo común. Se presenta en el 80% de las pacientes femeninas
y
13% de los masculinos en una defeco grafía (> a 1cm.) Entre mas grande es el
rectocele, mayor son los síntomas asociados, Dificultad en la evacuación, constipación
crónica, dolor rectal y perineal, sensación de masa y en ocasiones sangrado. La
necesidad de presión manual para ayudar a la evacuación o para vaciar el rectocele
es común en más del 50% de los pacientes.
El diagnostico se hace con un simple tacto rectal e inspección vaginal, pero debe
recordarse que rara vez es el rectocele un hallazgo aislado. La presencia cistocele,
peritoneocele, enterocele u otros prolapsos perineales y problemas funcionales deben
ser descartados antes de proponer la reparación quirúrgica del rectocele. Defeco
grafía,
Pruebas de función fisiológica del piso pélvico, pruebas de función urinaria, evaluación
del esfínter anorectal por ultrasonido y hasta la resonancia magnética han sido
propuestos antes de la cirugía.

Bibliografía:

Rectocele: Pathogenesis and surgical managment. Zbar AP, Linemann A, Fritsch H,


Beer-Gabel M, Pescatori M. Int J Colorectal Dis. (2003) 18:369-384.
Evaluation and Treatment of Women with rectocele. Cundiff GW, Fenner D, Obstetrics
and Ginecology 104(6): 1403-1416
Stapled transanal rectal resection to treat obstructed defecation caused by rectal
intussusseption and rectocele. Renzi A, Izzo D, Di Sanrno (2—6) 21:661-667
Rectocele repair using biomaterial augmentation. Altman D, Melgren A, Zetterstrom J.
Obstet Gynecol (2005) 60(11)753-760.

69.- Se trata de femenino de 27 años, es atendida en consulta por secreción láctea


bilateral hace 4 meses y menstruaciones cada 21 días, en escasa cantidad desde
hace 6 meses. antecedentes: hace 6 meses padece gastritis tratada con cimetidina
durante 2 meses y posteriormente ha tomado el medicamento de forma irregular.
a.g.o.: menarca 13 años, ritmo 30/7, núbil. e.f.: talla 1.63, peso 54 kg. al efectuar
compresión en glándulas mamarias se produce salida de secreción lactea bilateral,
resto normal.

El efecto colateral más probable que se puede presentar esta paciente es:

a) Esterilidad primaria.
b) Hipotensión refractaria
c) Dismenorrea.
d) Amenorrea secundaria

EFECTOS DE HIPERPROLACTINEMIA
Una secreción descontrolada de prolactina o hiperprolactinemia, puede provocar
anovulación y amenorrea secundaria de aparición espontánea, como en el síndrome
de Forbes-Albright, causada por tumor o también el síndrome de Chiari-Frommel que
cursa con galactorrea persistente después del embarazo debido a un trastorno
funcional.
Haciendo una revisión histórica de este padecimiento, Greenblatt menciona a la
galactorrea como síntoma principal, y ha sido reportada desde antiguas leyendas
mitológicas, la Biblia en el libro de Jacob, menciona un caso de galactorrea patológica,
Chiari en 1855 describió un cuadro de lactancia persistente y amenorrea del posparto
y así muchos otros reportes han sido publicados.
En el síndrome de galactorrea-amenorrea, cuadro conocido también como de Argonz y
Del Castillo, es de utilidad diagnóstica, conocer los niveles basales de prolactina,
cuando se sospecha de tumores productores de prolactina (causa número 1 de
tumores pituitarios) que llegan a producir hasta más de 100 ng/mL., llamados
adenomas que es una neoplasia endocrina múltiple tipo 1, productores de prolactina
(el 30% de ellos, Melmed 1986), prolactinomas, micro adenoma de la hipófisis (tamaño
menor a 10 mm), macro adenoma (tamaño mayor a 10 mm), etc., de por sí la
presencia de galactorrea puede ser un signo de un proceso maligno subyacente. En
reportes de otros autores como son Sakiyama y Quan
(Obstet.Gynecol.Surv.38:689,1983) este padecimiento sería del 40 al 50% de todas las
formas de lactación no puerperal.
El síndrome de galactorrea-amenorrea es un padecimiento que se presenta en
mujeres jóvenes en sus años reproductivos, puede estar asociado al embarazo o no,
la amenorrea se sospecha por un mecanismo de la prolactina periféricamente en el
ovario, por competencia de los receptores ováricos dirigidos a las gonadotrofinas,
siendo refractario a la estimulación exógena, por lo que en algunas mujeres que dan
lactación, la prolactina promueve la falta de reacción ovárica, mas este hecho no
confiere a la lactación una seguridad de anticoncepción. Los períodos menstruales y la
fertilidad vuelven al extirpar el tumor mediante hipofisectomía transesfenoidal. Cuando
se presenta recurrencia es recomendable aplicar bromocriptina incluso también para
aquellas en la que la cirugía no sea viable o que la causa no sea un tumor.
De los pacientes con acromegalia activa el 20 al 40% de ellos cursan con
hiperprolactinemia.
Cuando la hiperprolactinemia es idiopática se pueden presentar niveles hasta de 950
ng/mL.
Otra de las condiciones que puede provocar la hiperprolactinemia es la supresión de la
liberación pulsátil de la hormona luteinizante (LH) interfiriendo con el mecanismo de
retroalimentación positiva de estrógenos en la liberación de LH modificando la
secreción cíclica hipotalámica de la hormona liberadora de hormona luteinizante
(LRH), habiendo mencionado esto se puede comprender mejor el papel de la
prolactina en padecimientos como la anovulación, amenorrea, oligomenorrea, fase
lútea corta, síndrome de ovarios poliquísticos, esterilidad, etc.
Es importante mencionar que no siempre que exista galactorrea la prolactina estará en
niveles elevados, por lo que debe realizarse una investigación más cuidadosa.
Algunos pacientes con hiperprolactinemia idiopática (16%) presentan auto anticuerpos
anti-prolactina, de ellos en el 4% se indujo la hiperprolactinemia por drogas, el 3%
presentaron prolactinoma y otro 3% asociado a otras causas. Estos pacientes
usualmente no tienen sintomatología como amenorrea o galactorrea.
Debe quedar bien claro que galactorrea e hiperprolactinemia no son sinónimos, es
probable que el inicio haya sido un aumento transitorio de la hormona dando por
resultado el síntoma que se mantiene aún desaparecido el estímulo que le dio origen.
Por otro lado una deficiente secreción puede originarse por un infarto hipofisiario como
en el síndrome de Sheehan (insuficiencia de la hipófisis anterior), que se advierte en el
período posparto, como signo precoz y valioso de hipopituitarismo que puede ser
parcial o total, o bien una deficiencia de secreción de prolactina aislada sin
manifestaciones clínicas

70.- Paciente femenino de 17 años que acude al servicio de urgencias por sangrado
vaginal, en la ecografía se observa un saco gestacional de aproximadas 7 semanas,
sin presencia de actividad cardíaca embrionaria, localizado a nivel infundibular. La
paciente está hemodinamicamente estable. El tratamiento de primera elección en la
paciente es:

a) Metrotexate a altas dosis durante un mes.


b) Expresión tubárica o salpingostomía por laparoscopia.
c) Actitud expectante.
d) Laparotomía urgente y anexectomía.

Con estabilidad hemodinámica:


a). El dolor será más intenso y puede irradiarse a otros sitios, a todo el abdomen o al
hombro (signo de Laffon).
b). La hemorragia, generalmente, no variará.
c). la palpación de la tumoración anexial es muy dolorosa.
d). Al examen, el fondo del saco de Douglas puede estar abombado o ser muy
doloroso.
e). Cuando se sospeche la rotura, no debe diferirse , confirmación por punción del
fondo del saco de Douglas o del abdomen. La punción también puede realizarse en
pacientes sin evidencias de complicación para poder asegurar que no existe un
hemoperitoneo pequeño o de poco tiempo de evolución.
f). En dependencia del tiempo de evolución del hemoperitoneo, se encontrarán signos
de irritación peritoneal e intestinal por la sangre libre.
g). Ante la sospecha o el diagnóstico presuntivo de embarazo ectópico, en una
paciente estable, debe realizarse una laparoscopia confirmativa que puede ser
seguida de cirugía laparoscópica o de una laparotomía para la operación
convencional.

EMBARAZO ECTÓPICO

SITIOS DE
IMPLANTACIÓN

Ampular
Itsmica
Intersticial
Infundibular
EMBARAZO ECTÓPICO
TRATAMIENTO CON METOTREXATO
 Mayor éxito:

 Embarazo menor a 6 SDG


 Masa tubaria menor de 3.5cm
 Feto sin latido cardíaco

Teran JMetotrexate y conducta expectante en el embarazo ectópico no roto


Rev Obstet Ginecol Venez.2002:62:3

EMBARAZO ECTÓPICO

TRATAMIENTO QUIRÚRGICO

 Salpingocentesis
 Expresión de las
fimbrias
 Salpingectomía
 Salpingostomia
lineal
 Resección
segmentaria

Hajenius PJ, Mol BWJ, Bossujt PMM, Ankum WN, Vander Veen F. Intervenciones para el embarazo ectópico tubarico
En: The Cochrane Library Isue 1; Oxford Update Software.

71.- Se trata de mujer de 18 años, acude al servicio de urgencias por presentar


sangrado transvaginal. Antecedentes: G2 - P1, cursa con embarazo de 9 semanas de
gestación. E.F. Sangrado transvaginal moderado, rojo rutilante. , cérvix posterior,
largo, blando, orificio cervical externo dehiscente, dolor a la movilización cervical,
rebote negativo. TA 110/70 MMHG FC 92X´FR 19 X´T 37 oC. Se realiza USG el cual
reporta saco gestacional con embarazo de 8.1 SDG, embrión sin latido cardiaco, la
conducta terapéutica más apropiada es:

a) legrado intrauterino
b) indometacina rectal por tres dosis
c) repetir ultrasonido en dos semanas
d) Laparotomía exploradora de urgencia
Si el embarazo ha sido diagnosticado como no viable, la mayoría de los médicos le
darán la opción de esperar a ver si el cuerpo va a sufrir un aborto natural o de
someterse a un procedimiento de dilatación y curetaje (LUI).
Current Obstetric & Gynecologic Diagnoses & Treatment, Ninth Ed., DeCherney, Alan
H., et al, Ch 8, 14 Williams Obstetrics Twenty Second Ed. Cunningham, F Gary, et al,
Ch 3 eMedicine, http://www.emedicine.com/ Creighton University Medical Center,
http://medschool.creighton.edu/medicine/departments/radiology/

72.- Mujer de 18 años diagnosticada de embrazo ectópico mediante ecografía que


también muestra gran cantidad de líquido libre en Douglas, con mal estado general.
No tiene hijos y conserva la trompa contralateral en perfecto estado. El tratamiento de
lección más apropiado para esta paciente es:

a) Expectante y determinación del B-HCG.


b) Salpingectomía.
c) Legrado de cavidad uterina.
d) Alta y revisión en una semana

SANGRADOS 1era MITAD EMBARAZO


EMBARAZO ECTOPICO – Tratamiento

• Quirúrgico.
– Salpingostomía lineal.
– Salpingectomía total.
– Ordeñamiento fimbrial.
– Legrado uterino instrumental post-laparoscópico.
(Reacción deciduoide)

• Médico.
– Metrotexate y Mifepristona (RU 486)

73.- Femenino de 39 años con embarazo de 32 semanas. Ingresa al servicio de


urgencias por presentar cuadro de 1 día de evolución por sangrado transvaginal leve,
rojo brillante, sin contractilidad uterina. A.G.O.: G-3, P-0, C-2. E.F.: IMC: 30 kg/m2,
F.U. de 28 cm, fcf presente, especuloscopía cérvix cerrado con huellas de sangrado.
USG reporta producto único vivo con fetometría normal, líquido amniótico normal y
placenta anterior que cubre parcialmente el orificio cervical interno.
El factor de riesgo principal para ésta paciente es:
a) cirugías uterinas previas
b) multiparidad
c) obesidad
d) edad avanzada

PLACENTA PREVIA
DEFINICIÓN:
Es cuando la placenta se implanta sobre o muy cerca del orificio cervical interno y una
parte de la placenta precede a la parte fetal que se presenta.
INCIDENCIA:
Esta es difícil determinar ya que muchos casos pasan desapercibidos, sobre todo
cuando ocurren los abortos en embarazos tempranos.
La prevalencia varía de 1 en 100 a 1 en 850 nacidos vivos, pero solo el 20% total.
Etiología:
Edad avanzada, multíparas, paciente con cesáreas previas, paciente con aborto de
repetición, esto debido a las gestaciones previas.
TIPOS:
Inserción baja.- Es cuando el borde placentario se encuentra en el segmento inferior a
menos de 6 cm del orifico cervical interno.
Marginal.- Es cuando el borde placentario alcanza los márgenes del orificio cervical
interno.
Parcial.- Es esta la placenta cubre parcialmente el orificio cervical interno.
Total.- La placenta cubre la totalidad del orificio cervical interno aún con dilatación
cervical avanzada.
DIAGNOSTICO:
La característica es el STV de aparición brusca en forma indolora en el segundo o
tercer trimestre. Frecuentemente hay ausencia de dolor a actividad uterina que son
parámetros para hacer el diagnóstico.
La mayor incidencia de sangrado aparece a las 33-34 sdg.
ESTUDIOS DE GABINETE:
El estudio mas utilizado es la ultrasonografía obstétrica.
TRATAMIENTO:
El manejo va a depender de factores como son:
Edad gestacional, magnitud del sangrado, si hay trabajo de parto, variedad de
placenta previa y complicaciones materna.
BIBLIOGRAFÍA:
1.- Waxler P, Gottesfeld KR. Early diagnosis of placenta previa. Obstet Gynecol
1979;54:231-32.
2.- Cabrero-Roura L. Riesgo elevado obstétrico. Ed. Masson 1996; pp; 109-118.
3.- Patrick J, Placenta Previa, Clinical Obst and Gynecology 1990;33(3): 414-421.
4.- Chapman M, Furtenes ET, Significance of ultrasound in location of placenta in early
pregnancy Br J Obst Gynecol 197;86: 846.
74.- Mujer de 23 años diagnosticada de E. Ectópico a nivel ampular, con saco
gestacional menor de 3 cm., sin actividad cardíaca embrionaria, asintomática y con
niveles de B-HCG menores para su edad gestacional. El tratamiento más indicado es:

a) Salpingocentesis
b) Resección segmentaria
c) Tratamiento médico con Metrotexate.
d) Histerectomía total con doble anexectomía.

TRATAMIENTO

 METOTREXATE
 Ácido 4 amino 10 metil folínico, antagonista del ácido fólico
 Glucosa hiperosmolar
 Prostaglandina F2a
 Actinomicina D
 Mifespristona

TRATAMIENTO CON METOTREXATO


 Mayor éxito:

 Embarazo menor a 6 SDG


 Masa tubaria menor de 3.5cm
 Feto sin latido cardíaco

PROTOCOLO CON DOSIS ÚNICA DE METOTREXATE

Día 0 hGC, Biometría hemática, transaminasas, creatinina, grupo sanguíneo


Día 1 hGC
Día 4 hGC
Día 7 hGC

 Disminución de hCG <15%. Segunda dosis


 Si la hCG declina seguirla semanalmente
 Si la hCG está en meseta o en ascenso, segunda dosis
 El raspado endometrial se realiza sólo en pacientes con hCG <2000 mIU/mL al
momento de comenzar el tratamiento

Graczykowski JW, Mishell DR. Methotrexate prophylaxis for persistent ectopic


pregnancy after conservative treatment by salpingostomy. Obstet Gynecol.

75.- Femenino de 32 años segunda gesta a término sin anormalidades en el


transcurso de éste. Inicia trabajo de parto de forma espontánea, con evolución normal
hasta que se rompe la bolsa, con una dilatación de 4 cm. A partir de entonces,
comienza con hemorragia de sangre roja, en moderada cantidad y aparecen signos de
sufrimiento fetal agudo. El estado general de la mujer es bueno y la dinámica uterina
es normal. Este cuadro corresponde a:
a) Abruptio placentae.
b) Rotura uterina.
c) Placenta previa central
d) Rotura de vasa previa.

La vasa previa es una condición de alto riesgo obstétrico en la cual vasos fetales o
placentarios cruzan el segmento uterino por debajo de presentación.
Estos vasos están desprotegidos de gelatina de Wharton o tejido placentario, lo que
los hace altamente vulnerables y susceptibles de ruptura o laceraciones en cualquier
período del embarazo, principalmente en el momento del parto . También es frecuente
la compresión de estos vasos, especialmente durante el tercer trimestre de la
gestación, lo que puede condicionar asfixia y muerte fetal.

Esta condición ocurre como resultado de que vasos velamentosos cruzan por el
segmento uterino debido a una inserción velamentosa del cordón, situación en la cual
el cordón umbilical se inserta en las membranas ovulares en vez del tejido placentario
(vasa previa tipo I), o por el cruce de vasos fetalesentre uno o más lóbulos accesorios
de la placenta (vasa previa tipo II) (Figura 1).

Figura 1. A, Vasa previa tipo I, debida a inserción velamentosa.


B, Vasa previa tipo II, debida a cotiledones aberrrantes. Reproducido de Daly-Jones y
cols. Ultrasound 2008.

Vasa previa se puede presentar si existe alguna (o ninguna) de las siguientes


condiciones:
placenta baja (que puede ser causa de abortos previos seguidos por legrado o por
operaciones uterinas, que provocan cicatrices en el útero), placenta bilobada o de
lóbulo succensuriado, embarazos resultado de fertilización in vitro, o embarazos
múltiples (5-6). El sangrado por vasa previa no es doloroso. Otros sangrados por
complicaciones o por nacimiento no necesariamente son sin dolor.

Referencias
2. Oyalesse KO, Turner M, Less C, Campbell S. Vasa previa: an avoidable obstetric
tragedy. Obstet Gynecol Surv 1999; 54: 138-145.
3. Sepúlveda W, Sebire NJ, Harris R, Nyberg DA. The placenta, umbilical cord, and
membranas. In Diagnostic Imaging of Fetal Anomalies, Nyberg DA, MaGahan JP,
Pretorius DH, Pilu G (eds). Philadelphia, PA: Lippicont Williams & Wilkins 2003, 85-
132.
4. Daly-Jones E, John A, Leahy A, McKenna C, Sepulveda W. Vasa praevia; a
preventable tragedy. Ultrasound 2006; 16: 8-14.
5. Derbala Y, Grochal F, Jeanty P. Vasa previa. J Prenat Med 2007; 1: 2-13.
6. Fung TY, Lau TK. Poor perinatal outcome associated with vasa previa. It is
preventable? A report of three cases and review of the literature. Ultrasound Obstet
Gynecol 1998; 12: 430-433.

76.- Una mujer de 33 años de edad G4 P3, acude a consulta en busca de información
acerca de los métodos anticonceptivos. No tiene APP de importancia, ni toma
medicamentos. Ha mantenido una relación monógama con su pareja por los últimos 9
años. No quiere volver a embarazarse, pero se niega a que se le practique una
salpingoclasia. Su médico recomienda un DIU. Con este método anticonceptivo
existe mayor probabilidad de:

a) Amenorrea
b) Embarazo ectópico
c) Embarazo intrauterino
d) Aumento de peso

Ventajas del DIU: Sumamente eficaz. No se requieren suministros constantes.


Fácil de usar. No interfiere con el acto sexual. La fertilidad regresa pronto.

Desventajas del DIU: PRESCRIPCIÓN MÉDICA para iniciar y discontinuar el uso.

Puede causar dolor o molestia leves durante los procedimientos de inserción y


extracción. Puede expulsarse. No protege contra las ETS / VIH

Efectos secundarios: Dolor o calambres durante la menstruación.

• Sangrado menstrual prolongado y abundante.

Dolor en la parte baja del abdomen, flujo vaginal, fiebre Embarzo ectópico
DIU 2 por cada 1000 usuarias

ETIOLOGIA
III.1 Factores que dificultan el camino del huevo. Este se implanta allí donde se
encuentra en el 6º-7º día postfecundación.
A/ Procesos inflamatorios, causa más frecuente de embarazo ectópico. Las salpingitis
deterioran la actividad ciliar y la motilidad tubárica. Hay otros procesos inflamatorios,
como, apendicitis que afectan secundariamente a las trompas, con producción de
adherencias y acodaduras.
B/ Alteraciones de la motilidad tubárica, como ocurre con la administración de
gestágenos a bajas dosis de forma continua (minipildora) o el empleo de
contracepción postcoital con estrógenos.
C/ DIUs. 9-17% de embarazos en portadoras de DIU son ectópicos, lo que se explica
porque el DIU protege más eficazmente frente a la gestación intrauterina (99’5% de
seguridad) que frente al tubárico (95% de seguridad) con lo que aumenta el riesgo
relativo de E.E.
D/ Tratamientos de esterilidad, sobre todo los quirúrgicos, cirugia conservadora de E.E
tubáricos, ligadura tubárica o técnicas de fecundación asistida (in vitro y transferencia
embrionaria intrauterina)
III.2 Factores que favorecen la nidación en la trompa: endometriosis tubárica, por un
cierto quimiotactismo del endometrio ectópico, así como, por la estenosis y
adherencias que se producen incluso con órganos vecinos.
III.3 Factores dependientes del propio huevo. Las anomalías genéticas del huevo,
mediante alteración en el momento de implantación, o quizá por un mayor volumen,
puede favorecer la anidación ectópica.

BIBLIOGRAFÍA EMBARAZO ECTOPICO


Cabero Roura, Ll y cols. Protocolos de Medicina Materno-fetal (Perinatología), 2ª
edición. Ed. Ergon, S.A. Madrid. 2000. 120-122.
Bajo Arenas, J.M, Castellanos Bolado, P. Embarazo ectópico. En: Manual de
asistencia a la patología obstétrica. Fabre E, Ed. INO: Zaragoza. 1997. 89 -107.
Carrera Macia, J.M y cols. En: Protocolos de obstetricia y Medicina perinatal del I. U.
Dexeus. 3ª Edición. Masson. Barcelona. 2000. 135-139.

77.- Se trata de mujer de 28 años, con tumor anexial de 6 cm líquido, dolor abdominal,
fiebre, leucorrea, con historia de cervicovaginitis de repetición y dispareunia crónica,
última menstruación hace una semana.
El manejo más adecuado para la paciente es:

a) Histerocopía
b) Colposcopía
c) Histerosalpingografía
d) Laparoscopía

CLASIFICACION LAPAROSCOPICA DE LA ENFERMEDAD PELVICA


INFLAMATORIA

El stándar de oro para el diagnóstico de EPI es la laparoscopia, ya que además de


visualizar directamente los órganos pélvicos, permite la toma de muestras para
estudios bacteriológicos.

Los criterios laparoscópicos para el diagnóstico de EPI, se describen en la tabla No.3.


Tabla No. 3

CLASIFICACION LAPAROSCOPICA DE LA EPI


Leve Eritema, edema, las trompas se mueven
libremente. No hay exudado purulento.
Moderada Eritema, edema mas marcado, material
purulento evidente. No hay movimiento libre
de las trompas. La fimbria puede no ser
evidente.
Severa Presencia de piosálpinx y/o absceso

EPI

Diagnóstico
Dolor abdominal bajo y/o dolor a la movilización cervical 
y anexial y:
Fiebre de 38º C o mayor
leucocitosis > 10, 500 mm3, eritrosedimentación y PCR elevada
Bacterias o leucocitos en liquido peritoneal
Masa anexial palpable o por USG
Laparoscopia
Síndrome de Fitz‐ Hugh‐Curtis

78.-Femenino de 25 años con antecedentes de G/2, P/1, C/1 acude al servicio de


consulta externa, refiere que presenta una secreción transvaginal bastante líquida, de
baja viscosidad, maloliente de color amarillo y gris espumoso. Esta entidad es propia
de infección por:

a) Cándida albicans
b) Gardenerella
c) Tricomonas
d) Gonococos

• El protozoario Thricomona Vaginalis es el responsable del 25% de las vaginitis.


• 20-50% de las mujeres cursan asintomáticas.
• La tricominiasis es predominantemente una infección transmitida sexualmente.
• Debe sospecharse de abuso sexual en caso de encontrar Trichomona en
pacientes pediátricos.

• DIAGNOSTICO:
• Flujo vaginal amarillo-verdoso con burbujas, muy fétido, irritación vulvo-vaginal,
disuria.
• El pH suele ser mayor de 4.5.
• Estudio en fresco en donde se observa al microscopio el organismo flagelado
característico.
• 75% se diagnostican con el estudio del Papanicolaou.

Referencias bibliográficas:
1. Secretaría de Salud. Norma Oficial Mexicana NOM -039-SSA2-2002, Para la
prevención y control de las infecciones de transmisión sexual. D.O.F. 19 de
Septiembre 2003.
2. Kettler H, White K, Hawkes S. Mapping the landscape for sexually transmitted
infections: key findings and recommendations. Geneva, TDR (TDR/STI/ IDE/04.1).
3. CDC. Trends in Reportable Sexually Transmitted Diseases in the United States.
CDC,
National Report. 2004
4. Distribución de los casos nuevos de enfermedades por mes Estados Unidos
Mexicanos
2004. Sistema Único de Información para la Vigilancia Epidemiológica/Dirección
General
de Epidemiología/SSA

79.- Se trata de paciente femenino de 19 años de edad, es atendida en la sala de


urgencias ginecoobstétricas, Antecedente: cursa embarazo de 38 SDG. Exploración
Física: Se en trabajo de parto. Repentinamente presenta sangrado profuso
transvaginal y dolor abdominal intenso. La causa más probable de la sintomatología de
esta paciente es:

a) Laceración vaginal por coito


b) Cervicitis
c) Placenta previa
d) Abruptio placentae

DESPRENDIMIENTO PREMATURO DE PLACENTA NORMOINSERTA (DPPNI):


Constituye la separación de la placenta de su área de inserción antes del 3º período
del parto. La mortalidad fetal es muy alta (superior al 15%) y la materna es tres veces
superior a la esperada.
La aparición frecuente de SFA, prematuridad, anemia, etc., hace que el número de
secuelas tanto sensitivas como motoras sea alto. Desde el punto de vista materno,
complicaciones secundarias a la hemorragia, a las alteraciones de la coagulación o
bien a la embolia pulmonar también tienen una tasa muy alta.
Desde la antigüedad se identificó al cuadro clínico caracterizado con la tríada
sintomática de hipertonía, metrorragia y muerte fetal, con pronóstico materno
comprometido. En 1775, Ricci diferenció la separación prematura de una placenta de
inserción normal, de aquella de inserción baja; a la primera llamó “hemorragia
accidental”, y a la segunda “hemorragia imprevisible”. Couvelaire introdujo el término
de “desprendimiento prematuro de placenta” y describió la aparición de la “apoplejía
uterina”

(útero de Couvelaire)
Obstetricia. Scwarcz, Sala, Duverges. 7ª edic. Edit. El Ateneo. (Biblioteca Fac.
Med. UNNE).

80.- Se trata de paciente femenino de 23 años acude al servicio de ginecología por


referir ciclos opso-menorreicos, desde el inicio de su menarquia, en los últimos 5
meses ha incrementado 15 Kg. de lo que pesaba habitualmente. Exploración Física:
acné facial importante, así como bigote.
La alteración fisiopatológica más común en esta paciente es:

a) anovulación y esterilidad
b) Hirsutismo
c) Amenorrea
d) Obesidad

Síndrome de Ovario Poliquístico (SOP) es uno de los más comunes trastornos


endocrinos que afectan a las mujeres alrededor del 5% al 10% de las mujeres en edad
reproductiva (12-45 años) y se piensa que es una de las principales causas de la
infertilidad femenina. Las características principales son la obesidad, anovulación
(dando lugar a la menstruación irregular) o amenorrea, acné, y las cantidades
excesivas o los efectos de androgénicos (masculinizantes) hormonas. Los síntomas y
la severidad del síndrome varían mucho entre las mujeres. Si bien las causas son
desconocidas, resistencia a la insulina, la diabetes y la obesidad están fuertemente
correlacionadas con el SOP.

Bulun SE, Adashi EY. The physiology and pathology of the female reporductive
axis. In: Kronenberg HM, Melmed S, Polonsky KS, Larsen PR, eds. Williams
Textbook of Endocrinology. 11th ed. Philadelphia, Pa: Saunders Elsevier;
2008:chap 16.

También podría gustarte